Тормозной путь физика: Тормозной путь — задание. Физика, 9 класс.

Содержание

Конспект урока по физике «Тормозной путь» (9 класс)

Тема: «Тормозной путь. Остановочный путь. Безопасность дорожного движения»

Цели урока:

  • Опираясь на знание физических законов, выработать осознанную необходимость соблюдения Правил дорожного движения. Познакомить учащихся с понятиями тормозной и остановочный путь.

  • Воспитывать законопослушность, ответственность за свою жизнь и жизнь людей, живущих рядом. Повторить основные знания по безопасному переходу улицы.

  • Развивать творческие способности, коммуникабельность

Ход урока:

  1. Орг. момент

Учитель: Часто на дорогах можно видеть перебегающего пешехода впереди идущего транспорта. По — видимому, пешеход не знает, что достаточно 30 секунд постоять на обочине, перекрестке перед красным сигналом светофора и для него загорится зеленый свет.Тормозной путь физика: Тормозной путь — задание. Физика, 9 класс.

Еще он не знает элементарных сведений о тормозном пути автомашин и не соблюдает Правила пешехода.

Сегодняшний урок посвящен взаимосвязи физики, математики и безопасности дорожного движения, мы познакомимся с понятиями тормозной путь и остановочный путь, повторим правила дорожного движения. Поэтому тема урока звучит так: «Тормозной путь. Остановочный путь. Безопасность дорожного движения» Слайд №1

Эпиграфом к нашему уроку я выбрала слова великого русского писателя А.П.Чехова «Солнце не всходит два раза в день, а жизнь не дается дважды…» Слайд №2

Проблема безопасности движения сложна и многогранна, вы каждый день идете или едете в школу, т.е. являетесь участниками дорожного движения.

  1. Актуализация знаний

Давайте посмотрим на фотографию, и наверное, тогда станет ясно, почему мы сегодня говорим о безопасности дорожного движения. Слайд № 3

Слайд № 4 Ежегодно в мире в результате ДТП погибают и получают ранения более 50 млн.Тормозной путь физика: Тормозной путь — задание. Физика, 9 класс. человек. Всемирная организация здравоохранения свидетельствует, что на долю ДТП более 30% смертельных исходов от всех несчастных случаев. В ХХ веке автомобиль стал причиной смерти около 30 млн. человек. В России потери, связанные с ДТП в несколько раз превышают ущерб от железнодорожных катастроф, пожаров и других видов несчастных случаев. Масштаб ДТП угрожает национальной безопасности.

Слайд № 5 Мы каждое утро выходим на улицу и становимся пешеходами. Анализ данных о количестве ДТП свидетельствует о том, что очень часто жертвами становятся пешеходы, в том числе и дети.

Исследования пешеходного движения показывают, что в зависимости от возраста и пола изменяется и скорость движения пешеходов:

Слайд № 6 Скорость передвижения пешеходов.

Таблица 1. Возраст и пол Скорость движения пешеходов

Дети 6-10 лет 1,11 м/с = 1,11X3600 : 1000 км/ч = 3,996 км/ч

Подростки 11-16 лет 1,59 м/с = 1.59X3600 : 1000 км/ч = 5,724 км/ч

Мужчины до 55 лет 1,62 м/с = 1.Тормозной путь физика: Тормозной путь — задание. Физика, 9 класс. 62 X3600: 1000 км/ч = 5,832 км/ч

Мужчины свыше 55лет 1,5 м/с = 1.5 X3600 : 1000 км/ч = 5,4 км/ч

Женщины до 55 лет 1,35 м/с = 1.35X3600 : 1000 км/ч = 4,86 км/ч

Женщины после 55 лет 1,29 м/с = 1.29X3600 : 1000 км/ч = 4,644км/ч

Слайд № 7 Пусть вам сообщили, что скорость мужчины 6 км/ч. Что означает это число?

Дорога — объект повышенной опасности. Помогают избежать опасных ситуаций на дорогах, конечно, дорожные знаки. Что означают эти знаки?

Слайд № 8 Вы ехали по городу и увидели знак ограничения скорости 40 км/ч, а в городе дует штормовой ветер, то есть где-то 25 – 30 м/с. Нарушает ли ветер правила дорожного движения? Что нужно нам сделать, чтобы сравнить эти две величины?

Выразим скорость движения ветра в других единицах (км/ч).

Давайте вспомним, что нужно для этого сделать: 30м/с = = 108 км/ч

Да. Нарушает.

Задача.

В начале участка шоссе стоит дорожный знак «30 км/ч». Нарушил ли правила движения водитель автомобиля, равномерно прошедшего участок дороги длиной 1,8 км за 4 мин? (Не нарушил, т.Тормозной путь физика: Тормозной путь — задание. Физика, 9 класс. к. скорость автомобиля была 27 км/ч).

Почему дети часто становятся жертвами ДТП?

Слайд № 9 1. При переходе дороги для того, чтобы иметь общее представление об окружающем пространстве, нужно осмотреться вокруг. Для того чтобы повернуть голову, ребёнку понадобится 4 секунды, а взрослому человеку всего лишь одну секунду.

Поэтому, искажённо оценив дорожную ситуацию, дети считают, что успеют перейти дорогу и попадают в ДТП.

Слайд № 10 2. Дети с искажением воспринимают звуки на дороге.

Подростки часто ходят по улицам города с наушниками, в которых звучит громкая музыка. И это очень существенно мешает восприятию окружающей действительности.

Слайд № 11 3. У ребёнка искажено восприятие размеров транспортного средства. Подростки до 13-14 лет видят только прямо. У них хорошо развито “тоннельное зрение”, а боковое зрение слабо фиксирует происходящее.

Слайд № 12 Решить задачу. Сколько секунд будет переходить ребёнок дорогу, если её ширина 6 метров и его скорость равна 1,2 м/с?

  1. Изучение нового материала

1.Тормозной путь физика: Тормозной путь — задание. Физика, 9 класс. Учитель. Ребята, помните – ни одна машина сразу остановиться не может. Знание того, сколько проедет автомобиль перед тем, как остановиться, какова дистанция безопасности, поможет избежать проблем на дороге.

1) Останавливаясь около заранее намеченного места, водитель выключает двигатель, и автомашина продолжает двигаться только по инерции. Затем водитель тормозит и плавно подводит машину к остановке.

2) Автомашина не может остановиться сразу, как только её затормозили. До полной остановки она проходит ещё некоторое расстояние – тормозной путь.

Слайд 13. Тормозной путь.

Определение 1. Тормозной путь — это расстояние, пройденное автомобилем с момента нажатия на педаль тормоза до полной остановки автомобиля.

Вопросы. От чего зависит тормозной путь автомобиля?

Тормозной путь зависит от многих факторов:

а) от силы сцепления колёс с землёй. Если дорога скользкая, шины стёрты, то сила сцепления колёс с землёй уменьшается и, наоборот, увеличивается, если дорога сухая, а шины новые.Тормозной путь физика: Тормозной путь — задание. Физика, 9 класс. В первом случае тормозной путь увеличивается, во втором сокращается.

б) от скорости автомобиля: чем больше скорость, тем он длиннее.

Слайд 14. Изменение тормозного пути.

Водитель едет со скоростью 25 км/ч, затем переходит на скорость 50 км/ч.

Как изменится путь торможения?

Слайд 15 Изменение тормозного пути от увеличения скорости

Казалось бы, что при удвоенной скорости тормозной путь тоже должен увеличиться вдвое. На самом деле он увеличивается в 4 раза. Если скорость увеличить в 3 раза, то тормозной путь увеличивается в 9 раз, а если в 4 раза – то в 16 раз.

Почему так происходит? Чтобы ответить на этот вопрос выведем формулу зависимости пути от скорости.

Слайд 16 — Если тело движется, то какой энергией оно обладает?

Кинетической энергией Ек = (1)

— Если тело обладает энергией и энергия уменьшается, значит тело выполняет работу, равную изменению этой энергии, т.е. А = (2)

— Какие силы действуют на автомобиль, если двигатель выключен?

Fтр = (3)

А = Fтрs = (4)

Приравняем 2 и 4 формулы

=

Задание командам из этой формулы вывести формулу пути.Тормозной путь физика: Тормозной путь — задание. Физика, 9 класс.

= =

Слайд 17 РЕШИМ ЗАДАЧУ: Определим тормозной путь при экстренном торможении.

Пешеход пересекает улицу в неположенном месте. Водитель замечает пешехода за 20 м и начинает экстренное торможение. Произойдёт ли аварии, если скорость авто 60 км/ч? Коэффициент трения 0,7.

60 км/ч = 16,7 м/с

Решение: рассчитаем тормозной путь по выведенной формуле:

=

S= 19,9 м.

В данной ситуации всё обошлось, а что было бы, если за 5 минут до этого прошёл дождь? Коэффициент трения = 0,5

S= 27,8 м. Машина собьёт человека.

Как изменяется тормозной путь в зависимости от скорости движения автомашины, показано в таблице.

в) от состояния дороги.

Числа в таблице приведены для сухой асфальтовой дороги.

Слайд 18. Тормозной путь.

Тормозной путь может увеличиться примерно на 30%, если дорога мокрая, и примерно в 3 раза, если дорога покрыта снегом, и в 5 раз, если асфальт покрыт ледяной коркой.Тормозной путь физика: Тормозной путь — задание. Физика, 9 класс.

В таблице указан тормозной путь по 1) мокрой дороге, 2) дороге, покрытой ледяной коркой.

Слайд 19. Тормозной путь.

г) от нагрузки и тяжести машины. Более тяжёлая машина (грузовик, автобус) имеет больший тормозной путь, чем, например, маленький “Москвич”.

д) от исправности тормозов, препятствий на пути и других условий.

  1. Закрепление.

Слайд 20. От чего зависит тормозной путь

а ) от силы сцепления колёс с землёй,

б) от скорости автомобиля,

в) от состояния дороги.

г) от нагрузки и тяжести машины

2. Введение понятия остановочный путь автомобиля.

Учитель. Ребята, вы увидели, что автомобиль нельзя остановить мгновенно. Для его остановки требуется определённое время, за которое он проходит некоторое расстояние.

Слайд 21 Остановочный путь.

Определение 2. Все расстояние, пройденное автомобилем с момента обнаружения опасности до полной остановки, называется остановочный путь автомобиля.Тормозной путь физика: Тормозной путь — задание. Физика, 9 класс.

Повторим Определение 1. Тормозной путь – это расстояние, пройденное автомобилем с момента нажатия на педаль тормоза до полной остановки автомобиля.

Слайд 22. Составляющие остановочного пути.

Остановочный путь состоит из двух частей: расстояние, пройденное автомобилем за время реакции водителя и тормозного пути.

Путь за время

реакции водителя Тормозной путь

Остановочный путь

Время реакции водителя.

Время реакции водителя колеблется от 0,5 с до 1,2 с. Что влияет на время реакции водителя?

Ответ. На время реакции водителя влияют личные качества: физическое состояние водителя, его возраст, водительский опыт.

Вопрос. Правильно ли, что чем больше скорость автомобиля, тем длиннее остановочный путь?

Ответ. Правильно. Чем больше скорость, тем больше инерция. Тормозной путь удлиняется, а, значит, удлиняется остановочный путь.

Слайд 23.

Задача.

Время реакции водителя на возникшую опасность составляет 0.Тормозной путь физика: Тормозной путь — задание. Физика, 9 класс. 8 с. Какой путь пройдет за это время автобус, если скорость его была 54 км/ч? (12м).

Слайд 24.

Задача.

Автомобиль движется со скоростью 40 км/ч. На расстоянии 15м у него возникает препятствие. Свернуть некуда. Реакция водителя 0,5 сек. Успеет ли водитель остановить машину? (Дорога сухая.)

Решение.

По таблице находим, что при скорости 40км/ч путь торможения по сухой дороге равен 10,4 м. До препятствия 15 м, значит, у водителя в запасе 15 м – 10,4 м = 4,6 м. Кажется, что автомашину можно успеть остановить. Но здесь мы не учли скорость реакции водителя. Если она составляет 0,5 сек, то при скорости 40км/ч автомашина за это время проедет

40 х 0,5 х 1000 : 3600 м = 5,6 м, а затем ещё 10,4 м до остановки, всего 16 м. А так как до препятствия 15 м, то водитель обязательно на него наедет.

Ответ. Не успеет.

Теперь становится ясно, ребята, какой опасности подвергается пешеход, пытающийся пересечь дорогу, по которой мчатся автомашины.Тормозной путь физика: Тормозной путь — задание. Физика, 9 класс.

Слайд 25. Запомни.

  • “Не перебегай улиц и дорог перед близко движущимся транспортом – это очень опасно для жизни”.

  • Ни одно даже самое важное дело не стоит вашей жизни.

Вопрос. Почему нельзя перебегать улицу перед близко идущим транспортом?

Слайд 26. Памятка.

Умей не только видеть, но и слышать улицу.

Обращай внимание на сигналы автомобиля (указатели поворота, заднего хода, тормоза)

Контролируй свои движения: поворот головы для осмотра дороги, остановку для пропуска автомобиля.

  1. Подведение итогов:

– Мы сегодня с вами не только повторили основные физические понятия, такие как скорость, тормозной путь, инерция и трение, но и рассмотрели их практическое применение, повторили правила дорожного  движения и дорожные знаки. Надеюсь, что данные знания помогут вам в жизни.

1. Если увеличить скорость транспорта вдвое, то потребуется вчетверо больший путь до его остановки, т.Тормозной путь физика: Тормозной путь — задание. Физика, 9 класс. е. тормозной путь увеличится в 4 раза, а время торможения – в 2 раза.

2. Чем больше масса транспортного средства, тем время торможения и тормозной путь больше, т.е. тем труднее изменить скорость автомобиля и, следовательно, тело более инертно.

3. Длина тормозного пути зависит от погодных условий: на мокрой, скользкой дороге сила сцепления колёс с дорогой уменьшается, а тормозной путь увеличивается.

4. Зависимость тормозного пути от тормозной системы, изношенности шин колёс, освещённости дороги и других факторов.

5. Для остановки транспорта требуется время и пространство: нельзя переходить дорогу перед близко идущим транспортом. Об этом следует помнить во избежание ДТП, как пешеходам, так и автомобилистам, велосипедистам и другим участникам движения.

Наиболее распространенные нарушения:

1) Переход перед близко идущим транспортом;

2) Внезапный выход на проезжую часть из-за стоящего транспорта;

3) Переход дороги вне пешеходного перехода;

4) Нарушение правил езды на велосипеде;

5) Игра на дороге.Тормозной путь физика: Тормозной путь — задание. Физика, 9 класс.

Скоро растает снег и многие из вас начнут кататься на велосипедах. Весна радует нас своим приходом. Голубое небо, яркое солнце, таяние снега — всё это поднимает настроение после долгой, суровой зимы. В связи с потеплением на дорогах появится и больше транспорта, что влияет на безопасность дорожного движения. Но, соблюдая установленные правила, мы тем самым оберегаем себя и окружающих от возможных неприятных последствий

(Выставление оценок активным ученикам)

Слайд 26. Домашнее задание:

  1. Составить кроссворд содержащий основные физические понятия и понятия из правил дорожного движения.

  2. Задача. На участке дороги, где установлен такой знак, водитель применил аварийное торможение. Инспектор обнаружил по следу колёс, что тормозной путь равен 12 м. Нарушил ли водитель правила, если коэффициент трения 0,6?

что это, значение, принцип работы

Тормозной путь автомобиля — это дистанция до полной остановки, которую успевает преодолеть машина с того момента, как водитель нажал педаль тормоза.Тормозной путь физика: Тормозной путь — задание. Физика, 9 класс. Важно понимать, что остановочный путь всегда больше, чем тормозной. Ведь он включает еще и расстояние, пройденное с того момента, как водитель обнаружил опасность и нажал на тормоза.

Как рассчитать тормозной путь

Длина пути рассчитывается по следующей формуле:

l = V2/(2µg), где

  • l — путь,пройденный автомобилем;

  • V — скорость авто;

  • µ — коэффициент, определяющий силу трения

  • g — ускорение свободного падения (9,8м/с2).

Скорость легко определяется по показаниям спидометра, а коэффициент трения покрышек на сухом асфальте колеблется в диапазоне 0,5-0,8.

Для приблизительных расчетов используется µ=0,7.

Для скорости 60 км/час (по системе СИ — 16,7 м/с) тормозной путь равен:

16,72/(2*0,7*9,8)=20,24 метров.

Столько проедет серийная машина с момента начала торможения.

Однако такое значение актуально лишь для условий, приближенных к идеальным.Тормозной путь физика: Тормозной путь — задание. Физика, 9 класс. При неравномерном срабатывании тормозов (цилиндров и колодок на каждом колесе) машина может потерять управляемость. Для восстановления контроля придется ослабить нажатие на тормоз. В этом случае тормозной путь будет значительно длиннее.

При расчете пройденного расстояния учитывается квадрат скорости. То есть, с ростом скорости тормозной путь резко удлиняется. При 80 км/час он составит уже 36 м, а на 120 км/час — 81 метр.

От чего зависит длина тормозного пути

Как видим, на тормозной путь влияют два параметра: скорость и коэффициент трения. Если скорость полностью зависит от действий водителя, то с трением все значительно сложнее. Давайте разберемся, какие факторы на расстояние, необходимое машине для остановки.

Состояние шин

Коэффициент сцепления (µ) зависит от следующих параметров:

Изношенный протектор сильно ухудшает торможение на мокрой, заснеженной или даже грязной дороге. Зато зависимость тормозного пути от температуры нелинейна.Тормозной путь физика: Тормозной путь — задание. Физика, 9 класс.

При низких температурах резина теряет эластичность и коэффициент трения уменьшается. Поэтому в холодное время года нужно использовать зимние шины независимо от того, как успешно дорожные службы справляются с уборкой снега. Зимой даже на чистом асфальте тормозной путь на зимней резине будет намного короче, чем на летней.

При высоких температурах резина становится слишком мягкой. При этом она интенсивно изнашивается и начинает легче скользит по асфальту. Поэтому летом быстрее остановиться получится на летней резине, которая сохраняет эластичность, но не «течет» подобно пластилину.

Дорожное покрытие

Коэффициент трения, который на сухом асфальте равен 0.7, меняется в зависимости от погодных условий:

  • 0,1 — гололед;

  • 0,2 — снежный накат;

  • 0,4 — мокрый асфальт.

В летнее время нужно остерегаться больших луж и грязи. Лужи могут вызвать эффект аквапланирования, при котором сцепление с дорогой будет даже хуже, чем на укатанном снегу.Тормозной путь физика: Тормозной путь — задание. Физика, 9 класс. Не менее опасна и грязь: тонкий слой мокрой глины, практически невидимый глазом, делает асфальт таким скользким, что на нем становится сложно просто устоять на ногах.

Антиблокировочная система

Как известно из школьного курса физики, сила трения скольжения всегда ниже, чем трения покоя. То есть, при торможении «юзом» тормозной путь больше. Этот эффект давно известен опытным водителями. Чтобы быстрее остановиться и не потерять управление на скользкой дороге, они используют «прерывистое» торможение. Метод заключается в том, что при блокировке колес водитель на мгновение отпускает педаль тормоза и тут же нажимает ее снова.

На большинстве современных серийных авто устанавливается электронная антиблокировочная система. Она контролирует вращение каждого колеса и снижает давление в тормозной магистрали при блокировке. В отличие от «прерывистого торможения», ABS контролирует каждый тормозной цилиндр в отдельности и ослабляет торможение только для заблокированных колес.Тормозной путь физика: Тормозной путь — задание. Физика, 9 класс. За счет этого удается достичь минимального тормозного пути на сухом асфальте, гололеде и мокрой дороге.

Однако антиблокировочная система не всегда позволяет остановить авто быстрее, чем торможение «юзом». На снегу и грязи она не позволяет протектору поглубже «зарыться» в дорогу. Особенно заметен эффект при использовании шипованной резины. Поэтому если вы хотите, чтобы шипы эффективно тормозили, «вгрызаясь» в снег, лед или грязь, ABS стоит отключить.

«Законы механического движения в дорожной безопасности»

Интегрированный урок для 9–10 классов по физике

Мотивация и целеполагание

Учитель: Внимательно посмотрите на ряд фотографий (рис. 1) и на основе ассоциаций, которые они вызывают, составьте небольшой рассказ. Предположите завершение рассказа, что может быть на четвёртой фотографии?


Рис. 1.

Учащиеся: Машина движется на большой скорости и приближается к пешеходному переходу, по которому идут пешеходы, машина не успевает затормозить и сбивает пешеходов.Тормозной путь физика: Тормозной путь — задание. Физика, 9 класс. Четвёртая фотография может демонстрировать результат ДТП.

Презентация, слайд 1, продолжение.


Рис. 2.

Учитель: Верно (рис. 2). Какую фразу, слово нужно сказать водителю на первой фотографии, чтобы предотвратить такую цепочку событий?

Учащиеся: Притормози! (рис. 3)


Рис.3.

Учитель демонстрирует обучающий видеоролик кампании «Притормози!» с сайта ГИБДД.

Учитель: Целью кампании «Притормози!» является снижение количества погибших в ДТП пешеходов, а также сокращение аварий, произошедших по причине неправильного выбора водителями дистанции движения, скоростного режима, нарушения проезда пешеходных переходов. Все мы являемся участниками дорожного движения. Что должен знать водитель и пешеход?

Учащиеся: Правила дорожного движения.Тормозной путь физика: Тормозной путь — задание. Физика, 9 класс.

Учитель: На чём, как вы думаете, основываются многие Правила дорожного движения?

Учащиеся выдвигают предположения, учитель подводит их к идее, что некоторые Правила дорожного движения основываются на законах физики.

Учитель: Какова будет цель нашего урока?

Учащиеся: Убедиться в необходимости соблюдения Правил дорожного движения, опираясь на знания физических основ движения.

Введение новых знаний

Учитель: Какие силы действуют на тело при движении?

Учащиеся: Сила тяги двигателя, сила трения, сила тяжести, сила реакции опоры.

Учитель: Изобразите эти силы на рисунке.

Учащиеся делают рисунок в тетради, для поддержки при затруднениях используется презентация (рис. 4).


Рис. 4.

Учитель: Что необходимо сделать водителю, движущемуся в автомобиле, при возникновении препятствия?

Учащиеся: Затормозить.

Учитель: Что делает водитель при торможении, опишите процесс. (Важно, чтобы учащиеся полностью описали этот процесс, чтобы в дальнейшем ввести понятие остановочного пути.)

Учащиеся: Водителю необходимо, оценив ситуацию, сбросить ногу с педали газа и перенести её на педаль тормоза. При этом сила тяги двигателя отключается, машина движется под действием силы трения.

Учитель: Какие силы действуют на тело при торможении? Нарисуйте эти силы.

Учащиеся: Сила трения, сила тяжести, сила реакции опоры.

Учитель: Как движется тело под действием этих сил?

Учащиеся: Равнозамедленно.

Учитель: Что такое тормозной путь автомобиля? Как рассчитать тормозной путь автомобиля? (Учитель подводит учащихся под вывод формулы, задавая наводящие вопросы.)

Учащиеся: Запишем второй закон Ньютона, спроецируем его на координатные оси. В результате получим формулу α= μg. Используя формулу разности квадратов скоростей, получаем


Риc. 5.

Учитель: Проанализируйте формулу и назовите, от чего зависит тормозной путь автомобиля?

Учащиеся: Только от скорости движения в момент торможения и коэффициента трения.

Учитель: Как изменится тормозной путь автомобиля при увеличении скорости в два раза? Сделайте вывод.

Учащиеся: Увеличится в четыре раза, поэтому чтобы уменьшить тормозной путь, нужно снизить скорость движения.

Учитель: От чего зависит коэффициент трения?

Учащиеся: От природы соприкасающихся тел и качества обработки поверхности.


Рис. 6. Таблица зависимости коэффициента трения от рода соприкасающихся поверхностей

Учитель: Сравните коэффициент трения шин при разных видах поверхностей (рис. 6). Сделайте вывод.

Учащиеся: Коэффициент трения зависит от вида поверхности, и при изменении типа дороги или погодных условий тормозной путь будет разным.

Отработка полученных знаний на практике

Учитель: Обратимся к сайту http://bezdtp.ru/campaigns/pritormozi/calculator.php, на котором представлен калькулятор тормозного пути.

С какой разрешённой скоростью можно двигаться в городе? Выберите расстояние, с которого наш водитель увидит пешеходов на пешеходном переходе и начнёт экстренное торможение.

Учащиеся: Не более 60 км/ч.

Учащиеся предлагают расстояние. На уроке было предложено 50 м.

Учитель: Используя данную программу, рассчитаем тормозной путь автомобиля, движущегося на скорости 60 км/ч при разных погодных условиях, если пешеход был замечен водителем на расстоянии 50 м, и посмотрим результат движения.

Учитель выбирает условия движения: сухой асфальт, мокрая дорога, укатанный снег, обледенелая дорога и демонстрирует результат на экране. (Пример на рис. 7, 8.) Данная программа наглядно показывает учащимся тормозной путь в зависимости: от погодных условий, скорости движения автомобиля, дистанции до препятствия.

Учитель подводит учащихся к тому, что при разных погодных условиях необходимо правильно выбирать скоростной режим. При наличии компьютера на группу или пару учащихся, можно дать
задание по группам: подобрать для каждой дороги максимальную скорость, при которой не будет совершен наезд на пешехода.


Рис. 7. Окно программы «Калькулятор тормозного пути»

Учитель: Какой вывод для себя как будущие водители вы сделали? А что вы как пешеходы должны знать, выходя на пешеходный переход?

Учащиеся: Нужно выбирать скоростной режим в зависимости от погодных условий. Пешеход должен знать, что тормозной путь автомобиля зависит от погодных условий, и учитывать это, быть внимательным, автомобиль не может остановиться сразу. Внимательно смотреть на вывески: «Осторожно, гололёд!», «Скользкая дорога».


Рис. 8. Расчёт тормозного пути

Учитель: Вспомним начало урока. Сразу ли водитель начинает тормозить? Какие действия он выполняет, увидев препятствие?

Учащиеся: Водителю необходимо, оценив ситуацию, сбросить ногу с педали газа и перенести её на педаль тормоза.

Учитель: Требуется ли на это время? Что делает автомобиль, пока водитель готовится к торможению?

Учащиеся: Автомобиль продолжает движение.

Учитель: Верно. Время реакции опытного водителя 0,7–0,8 с, если он готов к торможению, если водитель не готов, невнимателен, то время реакции составляет уже 1,5–1,9 с. Кроме того, автомобиль, как техническое устройство, имеет определённое время срабатывания тормозной системы — от 0,2 до 0,6 с. Что произойдёт с тормозным путем автомобиля, если учитывать все эти факторы?

Учащиеся: Он увеличится.

Учитель вводит понятие остановочного пути (рис. 9).


Рис. 9. Остановочный путь

Учитель: Ещё раз, используя калькулятор тормозного пути, рассчитаем тормозной путь автомобиля, движущегося на скорости 60 км/ч, если пешеход был замечен водителем на расстоянии 50 м. Тормозной путь получился 20 м, водитель не задел пешехода.
А теперь рассчитаем остановочный путь автомобиля.

Из результатов расчёта программы остановочный путь автомобиля на сухом асфальте составил уже 62 м. Что произошло с автомобилем?

Учащиеся: Он въехал на пешеходный переход.

Учитель: Водителей штрафуют за разговоры по мобильному телефону во время движения. Оправданно ли это с точки зрения физики?

Учащиеся: Да, разговаривая, водитель отвлекается, время его реакции увеличивается и остановочный путь увеличивается. Водителю необходимо быть внимательным, не отвлекаться, не разговаривать.

Учитель: А как должны вести себя пассажиры автомобиля?

Учащиеся: Не отвлекать водителя разговорами.

Учитель: Скажите, большая ли разница в скорости 20 км/ч? Вспомните, как ведут себя ваши папы и мамы за рулём. С какой скоростью они едут, если разрешено 60 км/ч? Почему, как вы думаете?

Учащиеся: Многие водители считают разницу в скорости 20 км/ч небольшой. Сейчас штрафуют за превышение скорости только свыше 20 км/ч.

Учитель: А оправданно ли такое превышение с точки зрения физики?

Используя программу, рассчитаем остановочный путь, например при 40 и 60 км/ч, если препятствие находится на расстоянии 60 м.

Запишем данные в тетрадь: S1=37 м (при 40 км/ч), S2= 62 м (при 60 км/ч). Сможет ли избежать ДТП второй водитель?

А если машины идут в потоке (рис. 10)? Что нужно соблюдать водителю, кроме скоростного режима? Что должен учитывать пешеход, выбегая на пешеходный переход?


Рис. 10. Изменение тормозного пути в зависимости от скорости при движении в колонне

Учащиеся: Нужно соблюдать дистанцию. Чем выше скорость, тем больше дистанция. Пешеход должен знать, что нельзя перебегать дорогу перед близко идущим транспортом, это может привести к цепному ДТП.

Учитель: Проанализируем ещё раз формулу тормозного пути. Зависит ли тормозной путь от массы автомобиля?

Учащиеся: Нет.

Учитель: А реально зависит?

Учащиеся: Да.

Учитель: А в чём здесь причина? Что в формуле в неявном виде зависит от массы автомобиля?

Учащиеся обычно затрудняются, поэтому учитель подводит к пониманию, что при резком торможении происходит разрушение резины, что приводит к уменьшению коэффициента трения, а степень разрушения зависит от массы автомобиля (рис. 13).


Рис. 11. Зависимость тормозного пути от массы автомобиля

Обобщение и систематизация

Учитель: Обобщим материал урока.

Работа в группах.


Рис. 12.

Задание 1. Сформулируйте.

  • Что должен знать и учитывать водитель? К чему может привести «мнимое» понимание своего главенства на дороге водителем? (Группы 1 ,3, 5.)
  • Что должен знать и учитывать пешеход? К чему может привести «мнимое» понимание своего главенства на дороге пешеходом? (Группы 2, 4, 6.)

Осуществляется работа в группах, после обсуждения учащиеся выступают с сообщениями.

Задание 2 (творческое). Разным группам (всего 4 группы) предлагается поставить себя на место:

  • Пешехода
  • Водителя
  • Представителя Госавтоинспекции
  • Правительства РФ

и разработать меры повышения безопасности на дорогах. Меры записать на листе формата A3 и потом их представить. Результат работы групп оценивает представитель ГИБДД.

Одна из групп должна предложить использование световозвращающих элементов на одежде пешехода, если этого не произошло, то необходимо подвести учащихся под эту мысль.


Рис. 13.

Учитель: Учёными американского Корнельского университета были проведены исследования по зависимости восприятия водителями пешеходов в различной одежде в тёмное время суток. Водитель видит пешеходов в обычной одежде на расстоянии 30 м, а с использованием световозвращателей — за 150 м. Установим судьбу двух пешеходов, которых в черте города видит водитель рядом с пешеходным переходом. Один имеет световозвращатели, второй — в обычной одежде. Остановочные пути для скорости
40 и 60 км/ч были записаны в тетради.

Учащиеся делают выводы.

Далее учитель демонстрирует видеоролик, посвящённый использованию световозвращателей.

Подведение итогов

Учитель: Какая цель была на уроке? Как вы считаете, достигли мы этой цели? Сделайте заключительный вывод на основе данной цели.

В качестве домашнего задания я предлагаю вам посетить официальный сайт ГИБДД http://www.gibdd.ru/

На странице http://www.gibdd.ru/news/federal/648365/ просмотреть вместе со своими родителями видеоролики, в которых на примере научных фактов рассказано, почему
перед пешеходными переходами тормозить необходимо заранее.

Решить задачи:

1. Время реакции водителя на возникшую опасность составляет в среднем 0,8 с. Какой путь пройдёт за это время автобус, если скорость его была 54 км/ч?

2. Пассажир движущегося автобуса отвлёк разговором внимание водителя на 5 с. Почему «Правилами дорожного движения» запрещено это делать? Какой путь пройдёт за это время автобус, если его скорость была 60 км/ч?

3. Успеет ли водитель начать торможение, если на расстоянии 4 м от него на дорогу неожиданно выбежал пешеход? Скорость машины 36 км/ч, время реакции водителя 1 с.

4. Мальчик играл с мячом на тротуаре. Неожиданно мяч выкатился на дорогу. Чтобы поймать мяч и вернуться с ним на тротуар, мальчику необходимо 7 с. Какой путь пройдёт за это время машина, движущаяся со скоростью 60 км/ч? Почему запрещается детям играть на дорогах или около них?

Примечание:

Презентацию и видеоролики, использованные в уроке, вы найдёте на нашем сайте www.dddgazeta.ru в разделе «Банк идей» — «Педагогам и воспитателям».

Автор: учитель физики и информатики Н.В. Фирюлина, МБОУ «Лицей», г. Кирово-Чепецк, Кировская область

Другие статьи по теме: Наглядные пособия / Методика работы

 

Проектно-исследовательская работа «Тормозной путь»

Государственное бюджетное профессиональное образовательное учреждение

«Кузнецкий многопрофильный колледж»

Проектно-исследовательская работа

  Тормозной путь.

Выполнил: студент группы 14Ф

Казаков А.

Руководитель работы: преподаватель физики

Мустакаева Г.Р.

Кузнецк,2020

Содержание

Введение…………………………………………………………………………………………….3

Теоретическая часть……………………………………………………………………………5

Сила трения…………………………………………………………………………………..5

Тормозной путь автомобиля…………………………………………………………..6

Сцепление – основа безопасного вождения…………………………………..7

Практическая часть……………………………………………………………………………..9

Исследование наличия транспортных средств среди преподавателей и студентов колледжа………………………………………………………………………..9

Результаты практического исследования………………………………………..10

Вывод…………………………………………………………………………………………….11

Заключение…………………………………………………………………………………………12

Список литературы……………………………………………………………………………..13

Приложения………………………………………………………………………………………..14

ВВЕДЕНИЕ.

Обоснование выбора темы проекта.Проблема.

Я студент колледжа. Как и многие мои сверстники увлекаюсь автомобилями.Жду того момента,когда получу водительское удостоверение и смогу приехать на учебу на автомобиле.Изучая тему «Трение» на уроке по физике,затронули понятие «Тормозной путь и факторы ,которые на него влияют». Меня заинтересовали вопросы:нужно ли нам учитывать тормозной путь когда мы пользуемся транспортом или переходим дорогу перед транспорт,почему нельзя переходить проезжую часть дороги перед близко идущим транспортом, какое расстояние до движущегося транспортного средства они считают безопасным, что ведёт к роковым ошибкам на дороге, приводящих к увеличению ДТП и травматизму?

Актуальность и значимость проекта. В нашей стране, в нашем городе с каждым годом происходит увеличение транспортных,в том числе и среди студентов нашего колледжа. С увеличение автотранспорта происходит увеличение количества аварий.Как правило,водители-новички часто попадают в аварию.Так и множество пешеходов игнорируют правила дорожного движени,переходя дорогу в неположенном месте.Это связано с многими факторами,в частности с незнанием физических законов и факторов влияющих на тормозной путь.

Цель проекта: исследование влияния физических факторов , влияющих на длину тормозного пути.

В соответствии с поставленной целью в работе определены основные задачи:

Провести теоретический анализ литературы по природе сил трения,тормозного пути,безопасного движения.

Выполнить экспериментов, подтверждающих зависимость тормозного пути от скорости и от дорожного покрытия.

Провести анкетирование и опрос о наличие транспортных средств,о знаниях правил дорожного движения,о факторах,приводящих к несчастным случаям среди преподавателей и студентов колледжа.    

Проанализировать результатов изучения и исследования.

Гипотеза. Тормозной путь зависит от скорости и от коэффициента сцепления шин с дорогой.

Практическая значимость состоит в применении  зависимости тормозного пути от скорости и  от коэффициента сцепления шин с дорогой.

Методы исследования:  Изучение литературы и других источников информации,наблюдение,эксперимент,опрос(анкетирование).

Предмет: безопасность на дороге.

Объект исследования: тормозной путь, как физический фактор безопасного поведения на дрогах.

2.Теоретическая часть.

2.1. Сила трения.

Трение это неотъемлемая часть нашей жизни,без него мы не смогли бы существовать.В некоторых случаях трение может быть полезным и вредным. Что такое сила трения? Классическое определение звучит так: сила трения – это сила, появляющаяся при соприкосновении двух тел во время движения и препятствующая этому самому движению. Иными словами, чем больше сила трения между телами, тем труднее их двигать относительно друг друга. Что же касается самой физической природы трения, то оно появляется как результат взаимодействия между атомами и молекулами тел, соприкасающихся между собой.

В зависимости от характера движения тел различают такие виды сил трения как:

Покоя. Сила трения покоя возникает при соприкосновении двух тел, которые, однако, не движутся относительно друг друга, и имеет нулевое значение.

Скольжения. Сила трения скольжения – наиболее классическая иллюстрация действия трения, возникает при скольжении тел относительно друг друга. На ее величину влияет масса тела (чем она больше, тем больше сила трения), характер поверхности (разумеется, при скольжении по льду сила трения будет в разы меньше чем при скольжении по земле).

Качения. Сила трения качения появляется, когда одно тело катится по поверхности другого, например, при езде на велосипеде или автомобиле. При качении сила трения гораздо меньше, чем при скольжении. Это опытным, эмпирическим путем установили еще те далекие наши предки, которые изобрели колесо – величайшее изобретение в истории науки и техники.

Сила трения скольжения определяется формулой:

Fтр= μN= μFдавл. (1)

μ — коэффициент трения, N — сила реакции опоры, Fдавл. — сила нормального давления

Что же касается самого трения то и оно бывает нескольких видов:

Сухое – проявляется при соприкосновении твердых поверхностей.

Вязкое, также подобное трение называют жидкостным, появляется при соприкосновении твердого тела c жидкостью либо газом. Например, на корабль, плывущий по воде, как и на поверхность воды, действует вязкое (жидкостное) трение. Сила вязкого трения обычно гораздо меньше силы сухого трения.

Смешанное, возникает, когда между поверхностями, которые соприкасаются, есть слой смазки

2.2 Тормозной путь автомобиля.

Сила трения это сила,возникающая при движении одного тела по поверхности другого и направлена в сторону,противоположную данному движению. Если на тело не действуют другие тела,то тело останавливается.Если тело(автомобиль) движется с большой скоростью,то после торможения оно не может остановиться и продолжает двигаться по инерции и проходит некоторый путь до остановки. Наименьшее расстояние до остановки от момента торможения называется дистанцией безопасности.Даже, если за рулем машины сидит профессиональный водитель, на дороге всегда может возникнуть ситуация, когда необходимо максимально быстро остановить транспортное средство:

внезапное появление на дороге человека или животного;

неисправность транспортного средства;

нарушение другим водителем правил дорожного движения, что приводит к созданию аварийной ситуации;

непредвиденные обстоятельства: неровность дорожного покрытия, препятствие (упавшее дерево, камень) и т.п.

Автомобиль нельзя мгоновенно остановить.Для остановки автомобиля водитель использует педаль тормоза, приводя в работу его тормозную систему. С того момента,как начнут действовать тормоза,он пройдет до полной остановки некоторое расстояние- тормозной путь(приложение 1).

Тормозной  путь  — это  путь, пройденный  автомобилем  от  начала  торможения  до  полной  остановки.

По второму закону Ньютона: Fтр=ma a= , где а-ускорение автомобиля;

по закону Амонтона — Кулона Fтр= μN= μmg (2)

Ускорение: а= ,

Путь: S= ,

S= (3)

где: S — тормозной путь; Vо — скорость движения автомобиля в момент начала торможения ; μ — коэффициент трения шины о дорогу; g — ускорение свободного падения(приложение 2).

Из приведенной формулы видно, что пройденный до остановки путь пропорционален квадрату начальной скорости. Если увеличить скорость вдвое, то потребуется вчетверо больший путь для остановки. Это следует иметь в виду водителям транспортных средств. Об этом полезно помнить и прохожим, пересекающим оживленную улицу. Из формулы видно, что тормозной путь зависит и от коэффициента сцепления шин с дорогой. Однако значение последнего может измениться в зависимости от вида и состояния дорожного покрытия, типа шин автомобиля и давления воздуха в них.

2.3 Сцепление – основа безопасного вождения.

Сила трения покоя – она же сила сцепления.

Сила сцепления определяется по формуле: Fтр= μN= μmg (2)

Под действием силы тяжести автомобиля шина деформируется, образуя так называемое пятно контакта. Пятно контакта шины с дорогой – точка взаимодействия покрышки с поверхностью. Авторезина под тяжестью машины несколько деформируется. Вся площадь шины, контактирующая с асфальтом и будет точкой взаимодействия.Пятно контакта – это след колеса на дороге, его опорная поверхность(приложение 3). Среди автолюбителей принято считать, что чем шире шина, тем больше площадь пятна контакта шины с дорогой и тем лучше сцепление с дорогой, тем короче тормозной путь, тем лучше управляемость машины. Это не так.

Давление шины на асфальт равно весу шины, деленному на площадь контакта:

P = = , где P- давление шины на дорогу, N = mg -вес шины.

Тогда отсюда можно выразить вес через давление:

N = PS

Теперь, если подставить эту формулу в закон Амонтона- Кулона, получим:

F = µPS =S (4)

Сцепление шины с дорогой — сила трения покоя, и она не зависит от ширины шины и площади пятна контакта, а зависит от материала шин. Чем выше сцепление шины с доргой,тем безопаснее езда (приложение 4).

3.Практическая часть.

3.1. Исследование наличия транспортных средств среди преподавателей и студентов колледжа.

Анкетирование проводилось в одном из учебных корпусов колледжа.В качестве респондентов выступали преподаватели и студенты. На вопросы анкеты ответили 400 опрошенных. В анкете были представлены следующие вопросы:

Наличие транспортного средства.(да/нет)

Автомобиль это средство передвижения?(да/нет)

Автомобиль это роскошь?(да/нет)

Автомобиль это средство повышенной опасности?(да/нет)

Нужно ли перебегать дорогу перед близко движущимся транспортом?Почему?(да,успею/нет,не успею)

Случалось ли вам на большой скорости экстренно тормозить?(да/нет)

Вам удавалось быстро остановиться?(да/нет)

Вы знаете что такое тормозной путь?(да/нет/определение)

От чего зависит тормозной путь?(от скорости/от коэффициента сцепления/от дорожного покрытия/от ширины шин)

В результате анкетирования были получены следующие данные

Таблица 1

Количество участников опроса

400

Наличие транспортного средства

247/153

Автомобиль – это средство передвижения?

247

Автомобиль – это роскошь?

40

Автомобиль – это средство повышенной опасности?

50

Нужно ли перебегать дорогу перед близко движущимся транспортом?Почему?

50/350

Случалось ли вам на большой скорости экстренно тормозить?

150/97

Вам удавалось быстро остановиться?

150

Что такое тормозной путь?

240/160

От чего зависит тормозной путь?

210/190

Вывод.Не все респонденты знакомы с понятиями как тормозной путь и от чего он зависит.

3.2. Теоретический расчет тормозного пути по разным условиям, в зависимости от типа дорожного покрытия.

Расчет производился по формуле S=кэ,

где кэ-тормозной коэффициент, Фс-коэффициент сцепления.

Значения коэффициента сцепления шин с дорогой для различных дорожных условий (ГОСТ 50597-2017)

Таблица 2

Тип покрытия

Состояние покрытия

сухое

мокрое

Асфальтобетон и цементобетон

0, 7…0 ,8

0, 35…0 ,45

Гравийное покрытие

0, 6…0 ,7

0, 3…0 ,4

Грунтовая дорога

0, 5…0 ,6

0, 2…0 ,4

Дорога, покрытая укатанным снегом

0, 2…0 ,3

Обледенелая дорога, лед

0, 1…0 ,2

Таблица 3

Скорость к началу торможения в км/час

Тормозной путь в м

сухой асфальт

мокрый асфальт

снежная дорога

обледенелая дорога

20

2,25

4,5

7,9

15,7

30

5,06

10

17,7

35,4

40

9

18

31

62

50

14,1

28,2

49,2

98

60

20,24

40,5

70,8

141,4

100

56.24

112,4

196,8

393

3.3.Результаты практического исследования.

Таблица 3

Скорость к началу торможения в км/час

Тормозной путь в м

сухой асфальт

мокрый асфальт

снежная дорога

обледенелая дорога

20

3,15

4,9

8,2

17,4

30

6,1

11,3

17,7

36,5

40

10,6

19,3

32

63,6

50

15,1

30,2

50,6

98,7

60

24,2

50,7

88,2

151,4

100

57,8

113,4

200,3

395,3

Вывод. По результатам теоретических расчетов и практического исследования пришли к выводу,что наиболее безопасным покрытием для движения транспорта является сухой асфальт,а наименее безопасным является лед и чем больше скорость и меньше коэффициент сцепления, тем длиннее тормозной путь.Наша гипотеза потвердилась: тормозной путь зависит от скорости и от коэффициента сцепления шин с дорогой(приложение 5 ).

Результаты практического исследования несколько отличаются от теоретических расчетов.Это говорит о том,что кроме типа и состояния дороги, на длину пути торможения влияют наклон дороги, тип и степень износа протектора шин.

ЗАКЛЮЧЕНИЕ.

Движущийся автомобиль на большой скорости не сможет мгновенно останавиться. Прежде чем остановиться, он пройдет некоторое расстояние.  Скорость.Это ключевой фактор. При этом имеется в виду не только скорость езды машины, но и скорость реакции водителя. Считается, что реакция у всех примерно одинаковая, но это не совсем так. Играет роль опыт вождения, состояние здоровья человека, употребление им медикаментов и т.д. Также, многие «лихачи» пренебрегают законом и отвлекаются на смартфоны за рулем, что, в итоге, может привести к катастрофическим последствиям.

Помните еще один важный момент. Если скорость автомобиля увеличивается в два раза, длина его тормозного пути растет в 4 раза! Здесь пропорция 1:1 не работает.

Дорожные обстоятельства.Несомненно, на длину тормозной линии влияет состояние дорожного покрытия. На обледенелой или мокрой трассе она может вырасти в разы. Но это далеко не все факторы. Следует также опасаться опавших листьев, на которых шины прекрасно скользят, трещин на покрытии, ям и так далее. Шины.Качество и состояние резины сильно влияют на длину тормозной линии. Зачастую, более дорогие шины обеспечивают лучшее сцепление авто с дорожным покрытием. Обратите внимание, если глубина протектора стерлась больше допустимого значения, то резина утрачивает способность отводить достаточное количество воды при движении по мокрой дороге. В итоге, вы можете столкнуться с такой неприятной штукой, как аквапланирование — когда машина теряет сцепление с дорогой и становится полностью неуправляемой. 

Многих аварий можно было бы избежать, если бы водители следовали золотому правилу — держи дистанцию. В работе мы выяснили, какую дистанцию нужно соблюдать для собственной безопасности и как определить нужную дистанцию

Теперь мы точно знаем, от чего зависит тормозной путь: от скорости и коэффициента сцепления шин с дорогой.

Соблюдайте правила дорожного движения.Не переходите дорогу в неположенном месте.Не перебегайте дорогу перед близко идущим транспортом.Чтобы изменить скорость нужно время.(приложение 6).

             

СПИСОК ЛИТЕРАТУРЫ.

Элементарный учебник физики: Учебное пособие. В 3-хт. /Под ред.Г.С.Ландсберга. Т.1 Механика. Молекулярная физика.М.:Наука, 1985, 218 с.

Иванов А.С., Проказа А.Т. Мир механики и техники: Кн. для учащихся. – М.: Просвещение, 1993.

Бытько Н.Д. Физика, ч.1 и 2. Механика. Молекулярная физика и теплота.М.: Высшая школа, 1972, 336 с.

В.А.Касьянов.  Физика 10 класс.- М.: Дрофа, 2003, 412 с.

http://www.zp-avto.ru/articles/a137/

http://www.zp-avto.ru/articles/a137/

http://carlines.ru/modules/Articles/article.php?storyid=21 

https://zen.yandex.ru/media/avtotachki/tormoznoi-put-avtomobilia-vse-chto-nujno-znat-5e53ffa9c6e52572332e8625

https://tires1.ru/pyatno-kontakta/

Приложение 1

Приложение 2

Приложение 3

Приложение 4

Приложение 5

Скорость при торможении 60 км/ч

Скорость при торможении 40 км/ч

Скорость при торможении 20 км/ч

Приложение 6

Калькулятор остановочного пути автомобиля • Механика • Онлайн-конвертеры единиц измерения

Калькулятор определяет остановочный путь автомобиля с момента обнаружения водителем опасности до момента полной остановки автомобиля, а также другие параметры, связанные с этим событием, в частности, время восприятия водителем сигнала о необходимости торможения, время реакции водителя, а также расстояние, которое прошел автомобиль во время этих событий. Калькулятор также определяет начальную скорость (скорость до начала торможения) по известной длине торможения (длины тормозного пути) с учетом дорожных условий. Как и все остальные калькуляторы, этот калькулятор не следует использовать в судебных процессах и при необходимости получения высокой точности.

Пример 1: Рассчитать расстояние, необходимое для остановки автомобиля, движущегося со скоростью 90 км/ч по мокрой горизонтальной дороге с асфальтобетонным покрытием (коэффициент трения μ = 0,4) если время восприятия водителя 0,5 с и время реакции водителя 0,7 с.

Пример 2: Рассчитать начальную скорость автомобиля, движущегося по дороге с мокрым асфальтобетонным покрытием (μ = 0.4), если длина тормозного пути равна 100 м. Автомобиль движется на спуске с уклоном 10%.

Калькулятор остановочного пути

Входные данные

Начальная скорость

v0м/скм/чфут/смиля/ч

Время восприятия опасности водителем

thpс

Время реакции водителя

thrс

Уклон

σградус%

Движение вверх Движение вниз

Состояние дороги

—Сухой асфальтМокрый асфальтПокрытый снегом асфальтПокрытый льдом асфальт

или Коэффициент трения

μ

Тип привода тормозов

—ПневматическийГидравлический

или Время срабатывания тормозной системы

tbrlс

Выходные данные

Угол крутизны уклона θ= °

Замедление a= м/с²

Время торможения tbr= с

Расстояние, которое проедет автомобиль во время восприятия водителем опасности Shp= м

Расстояние, которое проедет автомобиль во время реакции водителя на опасность Shr= м

Расстояние, которое проедет автомобиль за время задержки срабатывания тормоза Sbrl= м

Тормозной путь Sbr= м

Остановочный путь Sstop= м

Критический угол наклона для заданного коэффициента трения θcrit= °

Критический уклон для заданного коэффициента трения σcrit= %

Определения и формулы

Остановочный путь

Остановочный путь — это расстояние, которое проходит автомобиль с момента, когда водитель видит опасность, оценивает ее, принимает решение остановиться и нажимает на педаль тормоза и до момента полной остановки автомобиля. Это расстояние является суммой нескольких расстояний, которые проходит автомобиль в то время, как водитель принимает решение, срабатывают механизмы тормозной системы и происходит замедление движения до полной остановки.

где shr — расстояние, которое проедет автомобиль во время восприятия и оценки водителем ситуации, shr — расстояние, которое проедет автомобиль во время во время реакции водителя на ситуацию, sbrl — расстояние, которое проедет автомобиль во время задержки срабатывания тормозов, и sbr — тормозной путь.

Расстояние, которое пройдет автомобиль во время восприятия и оценки водителем ситуации

Расстояние человеческого восприятия ситуации — это расстояние, которое пройдет автомобиль в то время, пока водитель оценивает опасность и принимает решение уменьшить скорость и остановиться. Оно определяется по формуле

где shp расстояние человеческого восприятия в метрах, v скорость автомобиля в км/ч, thp — время человеческого восприятия в секундах и 1000/3600 — коэффициент преобразования километров в час в метры в секунду (1 километр равен 1000 метров и 1 час равен 3600 секундам).

Расстояние, которое пройдет автомобиль во время реакции водителя

Расстояние реакции водителя — это расстояние, которое пройдет автомобиль пока водитель выполняет решение остановить автомобиль после оценки опасности и принятия решения об остановке. Оно определяется по формуле

где shp — расстояние реакции водителя с метрах, v — скорость автомобиля в км/ч и thr — время реакции водителя в секундах.

Расстояние, которое пройдет автомобиль во время срабатывания тормозной системы

Расстояние, которое пройдет автомобиль во время срабатывания тормозной системы, зависит от типа тормозной системы, установленной на автомобиле. Почти на всех легковых автомобилях и малотоннажных грузовых автомобилях используются гидравлическая тормозная система. На большинстве большегрузных автомобилей используются тормоза с пневматическим приводом. Задержка срабатывания пневматических тормозов приблизительно равна 0,4 с, а гидравлических (жидкость несжимаема!) 0,1–0,2 с. Общая задержка срабатывания тормозной системы измеряется как время от момента нажатия на педаль тормоза, в течение которого замедление становится устойчивым. Оно состоит из задержки срабатывания тормозной системы и времени установления постоянной величины замедления движения. В тормозной системе с пневматическим приводом воздуху необходимо время, чтобы пройти по тормозным магистралям. С другой стороны, в гидравлическом приводе задержек практически не наблюдается, и он работает в два—пять раз быстрее, чем пневматический.

Расстояние, которое пройдет автомобиль во время срабатывания тормозной системы, определяется по формуле

где sbrl — расстояние в метрах, которое пройдет автомобиль во время срабатывания тормозной системы, v — скорость движения автомобиля в км/ч, tbrl — время срабатывания тормозной системы в секундах.

Замедление

Для упрощения расчетов предположим, что автомобиль движется с постоянным ускорением или замедлением, которое определяется по известной из курса элементарной физики формуле равноускоренного или равнозамедленного движения

где a — ускорение, v — начальная скорость, v0 — конечная скорость и t — время.

Тормозной путь автомобиля

Тормозной путь автомобиля — это расстояние, которое проходит автомобиль с момента полного нажатия на педаль тормоза до момента полной остановки. Это расстояние зависит от скорости автомобиля перед началом торможения и от коэффициента трения между шинами и дорожным покрытием. В этом калькуляторе мы не учитываем другие факторы, влияющие на тормозной путь, например, сопротивление качению шин или лобовое сопротивление воздуха

В результатах исследования1, в котором коэффициент трения определялся путем измерения замедления, определено, что антиблокировочная тормозная система (АБС) влияла на коэффициент трения таким образом: он увеличивается с увеличением скорости при использовании АБС и уменьшается, если АБС не используется. В этом исследовании также подтверждается, что на коэффициент трения между шинами и дорожным покрытием влияет температура и интенсивность дождя.

Вывод зависимости тормозного пути от скорости и трения с использованием второго закона Ньютона

Коэффициент трения определяется как отношения силы трения к силе нормального давления, прижимающей тело к опоре:

или

где Ffr — сила трения, μ коэффициент трения и Fnorm — сила реакции опоры.

Действующая на тело нормальная сила реакции опоры определяется как составляющая силы реакции, перпендикулярная к поверхности опоры тела. В простейшем случае, когда тело находится на плоской горизонтальной поверхности, нормальная сила равна весу этого тела:

где m — масса тела и g — ускорение свободного падения. Эта формула выведена из второго закона Ньютона:

В более сложном случае, если тело расположено на наклонной плоскости, нормальная сила рассчитывается как

где θ — угол наклона между плоскостью поверхности и горизонтальной плоскостью. В этом случае нормальная сила меньше веса тела. Случай наклонной поверхности мы рассмотрим чуть позже.

В случае же горизонтальной поверхности, если коэффициент трения между телом и поверхностью равен μ, то сила трения равна

В соответствии со вторым законом Ньютона, эта сила трения, приложенная к движущемуся телу (автомобилю) приводит к возникновению пропорционального ей замедления:

или

Теперь, в соответствии с уравнением ускоренного (замедленного) движения имеем

Из курса элементарной физики известно, что при равнозамедленном движении с постоянным замедлением, если конечная скорость равна нулю, то тормозной путь определяется уравнением

Это уравнение можно переписать в более удобной форме с использованием преобразования скорости в км/час в м/с:

Подставляя в это уравнение a = μg, получаем формулу тормозного пути:

где скорость v задается в км/час, а ускорение силы тяжести g в м/с².

Решая это уравнение относительно v, получаем:

Аналогичную формулу для определения тормозного пути можно получить с помощью энергетического метода.

Вывод зависимости тормозного пути от скорости и трения с помощью энергетического метода

Теоретическое значение тормозного пути можно найти, если определить работу по рассеиванию кинетической энергии автомобиля. Если автомобиль, движущийся со скоростью v, замедляет движение до полной остановки, работа тормозной системы Wb, требуемая для полного рассеяния кинетической энергии автомобиля Ek, равна этой энергии:

Кинетическая энергия движущегося автомобиля Ek определяется формулой

где m — масса автомобиля и v — скорость движения автомобиля перед началом торможения.

Работа Wb, выполненная тормозной системой, определяется как

где m — масса автомобиля, μ — коэффициент трения между шинами и дорожным покрытием, g — ускорение силы тяжести и sbr — тормозной путь, то есть расстояние, которое прошел автомобиль от начала торможения до полной остановки.

Теперь, с учетом того, что Ek = Wb, имеем:

или

Скорость автомобиля до начала торможения является наиболее важным фактором, влияющим на величину остановочного пути. Другими, менее важными, факторами, влияющими на остановочный путь, являются время оценки водителем ситуации, время реакции водителя, скорость работы тормозной системы автомобиля и состояние дороги.

Время торможения

Из курса элементарной физики известно, что средняя скорость при равноускоренном движении равна полусумме начальной и конечной скорости:

С учетом, что конечная скорость равна нулю, время торможения определяется в калькуляторе как

Движение вверх и вниз по уклону

Силы, действующие на автомобиль на уклоне: Fg — сила тяжести (вес автомобиля), Fgd — скатывающая вниз составляющая веса автомобиля, Ffr — сила трения, действующая параллельно поверхности дорожного полотна с уклоном, Fgn — нормальная составляющая веса автомобиля, направленная перпендикулярно поверхности дороги, и Fnr — сила реакции опоры, равная нормальной составляющей веса автомобиля.

Когда водитель нажимает на педаль тормоза, замедляющий движение автомобиль может быть представлен в виде тела на поверхности с углом наклона θ (см. рисунок выше). Для простоты мы будем рассматривать только две силы, действующие на автомобиль, находящийся на уклоне. Это вес автомобиля и сила трения. Автомобиль, движущийся с начальной скоростью, замедляет движение, если сила трения, действующая параллельно дорожному полотну, больше, чем скатывающая сила, являющаяся составляющей силы тяжести, которая также параллельна дорожному полотну. Если начальная скорость автомобиля равна нулю, он в этой ситуации остается на месте при условии, что угол уклона меньше критического (об этом — ниже).

В то время, как сила тяжести Fg стремится скатывать автомобиль вниз, сила трения Ffr сопротивляется этому движению. Чтобы автомобиль мог в этой ситуации остановиться, сила трения должна превышать скатывающую составляющую силы тяжести Fgd.

В то же время, если сила трения превышает скатывающую составляющую силы тяжести, автомобиль будет двигаться вниз с постоянным ускорением и его тормозная система будет неспособна его остановить. Это может произойти, если угол наклона (уклон) дорожного полотна слишком велик или коэффициент трения слишком мал (вспомним как ведет себя автомобиль с обычными шинами на уклоне, если он покрыт коркой льда!).

По определению коэффициента трения, можно записать уравнение для силы трения:

или

Скатывающая составляющая силы тяжести:

Результирующая сила Ftotal, действующая на автомобиль на уклоне:

или

Как мы уже отмечали, сила Ftotal должна быть направлена вверх, иначе автомобиль при движении вниз остановить невозможно. В соответствии со вторым законом Ньютона, ускорение (точнее, замедление) автомобиля, движущегося под действием силы Ftotal, определяется как

Подставляя ускорение в выведенную выше формулу тормозного пути, получаем:

Решая это уравнение для vpre-braking, получим:

Отметим еще раз, что в этих формулах g задается в м/с, v в км/ч и s в метрах. В нашем калькуляторе используются две последние формулы.

Припаркованные и движущиеся по ул. Дивисадеро в Сан-Франциско (Калифорния) автомобили. Уклон дорожного полотна в этом месте равен 31% или 17°.

Уклон

Величина уклона дороги (показателя крутизны склона) равна тангенсу угла плоскости дорожного покрытия к горизонтали. Он рассчитывается как отношение перпендикуляра, опущенного из точки на поверхность (превышения местности) к длине горизонтальной поверхности от начала склона до перпендикуляра (горизонтальному расстоянию). По определению уклона считается, что при движении вверх уклон является положительным, а при движении вниз уклон является отрицательным, когда превышение в действительности является понижением дороги. Уклон дороги σ выражают как угол наклона к горизонтали в градусах или как отношение в процентах. Например, подъёму 15 метров на 100 метров перемещения по горизонтали соответствует уклон, равный 0,15 или 15%. В этом калькуляторе мы используем уклон в процентах, определяемый по формуле

где Δh — превышение местности и d — проекция уклона на горизонталь (см. рисунок выше). Если известен уклон, то угол наклона можно определить по формуле

Критический угол

При увеличении угла наклона дорожного полотна выше определенного значения, называемого критическим углом, движущийся вниз автомобиль затормозить невозможно, так как действующая на него сила трения становится меньше скатывающей силы. Этот критический угол находится из условия

или

или

Из этой формулы можно найти критический угол для данного коэффициента трения, при котором автомобиль не сможет затормозить:

Уклон, выраженный в процентах, определяется по известному углу наклона таким образом:

Пример

В этом примере мы покажем, как использовать формулу для определения тормозного пути. Пусть автомобиль движется с начальной скоростью vpre-braking = 90 км/ч вниз по уклону σ = 5% по мокрому асфальту (коэффициент трения μ = 0,4). Нужно определить тормозной путь. Для расчетов используем выведенные выше формулы.

Особые случаи

Нажмите на соответствующую ссылку, чтобы посмотреть как работает калькулятор в особых режимах:

Литература

  1. Hartman, J 2014, Effects of velocity, temperature And rainfall on the friction coefficient of pneumatic tyres And bitumen roads, Doctor of Philosophy (PhD), Aerospace, Mechanical And Manufacturing Engineering, RMIT University PDF 48 MB
  2. Wikibooks. Fundamentals of Transportation

Автор статьи: Анатолий Золотков

Физика торможения — Kaminsky Driving School — LiveJournal

Друзья, в прошлом выпуске я утверждал, что тормозной путь авто не зависит от его массы. Большинство водителей считают, что зависит, и я объяснил, откуда берется это представление. В этой статья я докажу справедливость своего утверждения, прибегнув к физическим понятиям. Подчеркну, что речь идет о кратчайшем, экстренном, то есть минимально возможном тормозном пути. То есть о тормозном пути при торможении на грани блокировки колес. В современных машинах при таком торможении срабатывает АБС (антиблокировочная система тормозов), а классические машины либо срываются в «юз», либо остаются на грани «юза», в зависимости от действий водителя.

Сначала докажу это «на пальцах». Утяжеляя машину, с одной стороны, мы увеличиваем ее инертность и осложняем торможение. С другой стороны, мы сильнее прижимаем шины к дороге, увеличиваем сцепление шин с дорогой и повышаем тормозные возможности машины. Эти два эффекта компенсируют друг друга в равной степени, и, в конечном итоге, масса не влияет на длину тормозного пути.

Для интерсующихся, приведу физико-математическое доказательство (смотрите полную версию статьи на моем сайте: http://www.kaminsky.su/own/blog/23-blog/242-fizika-tormozhenija)

Тормозной путь зависит от скорости, качества шины и качества дороги.

Как видите, тормозной путь не зависит не только от массы автомобиля, он также не зависит от площади пятна контакта шины с дорогой, от размеров и материалов тормозных дисков и колодок, от числа поршней в тормозных суппортах и т.д. Почему? Потому что длина тормозного пути определяется моментом блокировки колес тормозами и перехода шин из качения в скольжение. Если это произошло – колеса заблокировались, то какая разница, сколько поршней давят на колодки, из какого материала они сделаны и какой радиус тормозного диска? Все эти ухищрения сделаны, чтобы снизить перегрев тормозов и сделать их пригодными для многократных интенсивных торможений. То есть, чтобы сохранить длину тормозного пути, сделать его стабильным в случае многих попыток торможения. Но сократить тормозной путь, задаваемый сцеплением шин с дорогой, никакая тормозная система не сможет.

Перераспределение веса машины, которое имеет место при торможении, высота центра тяжести машины, колея, колесная база — также не влияют на тормозной путь.

Выходит, и Жигули, и Ferrari затормозят с примерно одинаковым тормозным путем, если тормоза у всех исправны, а на колеса установлены одни и те же шины. Возможна разница за счет разного времени срабатывания тормозной системы, а также за счет разных алгоритмов торможения водителя и АБС. Но эта разница будет куда меньше по сравнению с тем, когда одни и те же Жигули (или Ferrari) будут тормозить сначала на Michelin, а потом на отечественной Каме.

Кстати, не стоит сравнивать тормозные пути седана и фуры. Это не корректно, поскольку там разные конструктивно тормоза (у грузовиков даже бывает не гидравлическая, а пневматическая тормозная система с огромной задержкой в срабатывании) и разного качества шины. Сравнивать нужно «яблоки с яблоками», то есть одну и ту же машину с разной степенью загрузки.

Однако, если время срабатывания тормозов у легковушки и фуры одинаково, и стоят одинаковые по составу шины, то тормозной путь отличаться не должен. Вот видео, которое подтверждает это:

http://www.youtube.com/watch?v=zWwK0eClp6Y&feature=player_embedded#at=113

В заключение скажу, что тормозной путь зависит от веса машины (не будем путать вес и массу), а также от массы прицепа без тормозов, от положения руля. Обо всем этом я расскажу в будущих выпусках.

Продолжение следует…

Тормозной путь — Падение и остановка — GCSE Physics (Single Science) Revision — Other

Чтобы быть безопасным водителем, вам необходимо понимать факторы, влияющие на тормозной путь автомобиля:

тормозной путь = расстояние мысли + тормозной путь

Дистанция мышления

Водителю требуется время, чтобы отреагировать на ситуацию и затормозить. Автомобиль продолжает движение в это время реакции. Расстояние мышления — это расстояние, пройденное за это время реакции.

Расстояние мышления увеличивается, если увеличивается время реакции. Это может произойти, если водитель:

  • устал
  • отвлекся
  • под воздействием алкоголя или других наркотиков

Расстояние мышления также увеличивается с увеличением скорости автомобиля (поскольку автомобиль будет двигаться дальше в течение времени реакции) .

Тормозной путь

Тормозной путь — это расстояние, необходимое для остановки после включения тормозов. Тормозной путь увеличивается, если:

  • тормоза или шины в плохом состоянии
  • плохие дорожные и погодные условия (например, обледенелая или влажная дорога)
  • автомобиль имеет большую массу (например, в нем больше людей)

тормозной путь также увеличивается с увеличением скорости автомобиля.

На этом графике сравнивается разница между дистанцией мышления и тормозным путем на разных скоростях:

Когда тормоза нажаты, работа выполняется за счет силы трения между тормозами и колесами. Это имеет следующие эффекты:

  • уменьшает кинетическую энергию транспортного средства (потому что его скорость уменьшается)
  • увеличивает температуру тормозов

GCSE PHYSICS — Как скорость влияет на тормозной путь автомобиля? — Как удвоение скорости влияет на тормозной путь автомобиля?

GCSE PHYSICS — Как скорость влияет на тормозной путь автомобиля? — Как удвоение скорости влияет на тормозной путь автомобиля? — ОБУЧЕНИЕ НАУКА.

gcsescience.com 29 gcsescience.com

Силы и движение

Тормозной путь автомобиля — Скорость.

Всего тормозной путь = расстояние мышления + торможение расстояние.

И мыслительная дистанция, и торможение расстояние
изменяются как скорость автомобиля меняется.

См. Также расчет силы нужно остановить движущуюся машину
используя кинетический энергия или импульс.


Как скорость влияет на торможение Расстояние до машины?

Торможение расстояние автомобиля увеличивается с увеличением скорости.
Два расчета ниже показывают, как
удвоение скорости изменяет торможение расстояние автомобиля.


Q1. Автомобиль движется со скоростью 10 РС.
При торможении автомобиль замедляется
и имеет постоянное отрицательное ускорение из -2 м / с 2 .
Какой у него тормозной путь?

A1. Узнай сколько времени машина взял на остановку.
Затем найдите среднюю скорость автомобиля.
Затем рассчитайте тормозной путь.

Используйте a = (v-u) ÷ t
, чтобы найти время (t) на сколько долго нужна машина, чтобы останавливаться.

а = — 2
v = 0
u = 10

t = (v-u) ÷ a
t = (0–10) ÷ -2
т = 10 ÷ 2

т = 5 секунд.

Для объекта с постоянным ускорением,
средняя скорость = (начальная скорость + конечная скорость) ÷ 2
= (10 + 0) ÷ 2
= 5 м / с.

As скорость = расстояние ÷ время
тогда расстояние = скорость x время

Тормозной путь автомобиля
= 5 x 5
= 25 м.


Q2. Одинаковый машина теперь движется с вдвое большим скорость при 20 м / с.
Когда тормоза затянуты,
у машины такое же постоянное отрицательное ускорение -2 м / с 2 .
Какое у него торможение расстояние?

A2. Используйте тот же метод, что и выше, a = (v-u) ÷ т

а = -2
v = 0
u = 20

t = (v-u) ÷ a
t = (0–20) ÷ -2
т = 20 ÷ 2

т = 10 секунд.

г. средняя скорость = (начальная скорость + конечная скорость) ÷ 2
= (20 + 0) ÷ 2
= 10 м / с.

С скорость = расстояние ÷ время
расстояние = скорость x время

Тормозной путь автомобиля
= 10 x 10
= 100 м.

Обратите внимание, что удвоение скорости автомобиля от 10 до 20 м / с
более чем вдвое увеличил торможение расстояние.
Фактически тормозной путь увеличивается x4, когда скорость идет вверх x2.
Это из-за
эффект скорости от кинетической энергии автомобиля.

Ссылки Силы и движение Расчеты Вопросы по пересмотру

gcsescience.com Викторина по физике Показатель Force Quiz gcsescience.com

Дом GCSE химия GCSE Физика

Авторские права © 2015 gcsescience.com. Все права защищены.

какие факторы влияют на тормозной путь тормозной путь скорость размышления скорость кинетическая энергия время реакции эксперименты торможение дорожного транспортного средства фрикционные тормоза igcse / gcse 9-1 Физика примечания к пересмотру

5. Время реакции и тормозной путь, например, дорожные транспортные средства и решение проблем с использованием уравнения 2-го закона Ньютона и расчетов кинетической энергии

Док Брауна Примечания к редакции школьной физики: физика GCSE, физика IGCSE, O level физика, ~ 8, 9 и 10 школьные курсы в США или эквивалентные для ~ 14-16 лет студенты-физики

Какая формула остановки расстояние? Какие факторы влияют на расстояние мышления?

Какие факторы влияют на тормозной путь? Какая связь между тормозной путь и кинетическая энергия? Можете ли вы придумать простой эксперимент, чтобы измерить время реакции?

Субиндекс этой страницы

(а) Введение — тормозной путь и скорость дорожная техника

б) Как рассчитать дистанцию ​​мышления и тормозной путь от скорости — графики времени

(в) Факторы, влияющие на расстояние мышления (следовательно, и тормозной путь)

г Факторы, влияющие на тормозной путь (следовательно, и тормозной путь)

д) Графический анализ тормозного пути, скорости и кинетической энергии движущегося автомобиля

(ж) Подробнее о физике торможение автомобиля и кинетическая энергия

(г) Проблемы здоровья и безопасности, связанные с столкновениями с участием автотранспортных средств и велосипедистов

(высота) Некоторые продвинутые расчеты тормозной силы и кинетической энергии

(i) Простая реакция время эксперименты


а) Введение — s расстояния до верхней части и скорость дорожная техника

Очевидно, что при вождении автомобиля нужно будьте внимательны к любым внезапным изменениям в вашей ситуации, особенно если вам нужно Аварийный тормоз для остановки.

В этой ситуации вы хотите остановить автомобиль (или любое другое дорожное транспортное средство) в кратчайшие сроки до произведите соответствующую аварийную остановку!

Это означает приложение максимальной силы на педаль тормоза.

Больше времени на реакцию и больше времени требуется для остановки , тем больше риск сбоя в объект на вашем пути.Время «думающей» реакции каждого на ситуацию Требование быстрой физической реакции отличается, хотя обычно в диапазоне От 0,2 до 0,8 секунды. В биологии вы, возможно, изучали нервная система, включая рефлекторную дугу.

Расстояние, необходимое для остановки дорожного транспортного средства в аварийной ситуации определяется по следующей формуле:

РАССТОЯНИЕ ОСТАНОВКИ = РАССТОЯНИЕ МЫШЛЕНИЯ + ТОРМОЗНОЕ РАССТОЯНИЕ

Расстояние мышления — это как далеко вы путешествуете во время вашей реакции, которое является временным интервалом от вас воспринимают опасность и начинают действовать e.грамм. затормозить.

Тормозной путь фактический расстояние, с которого вы путешествуете, когда вы впервые нажимаете на тормоза, до остановка.

Тормозной путь — это общий время, необходимое от первоначального зрительного стимула до фактической остановки движения.

В приведенной выше таблице приведены типичные или средние значения для обдумывания расстояния, тормозного пути и тормозного пути и цитируется из дорожного кодекса Великобритании буклет с инструкциями.

Вы можете видеть, что расстояние мышления довольно значительная часть общего тормозного пути, особенно на меньшие скорости, НО посмотрите, насколько резко общий тормозной путь увеличивается с увеличением скорости.

Эти значения следует удвоить для мокрые дороги и умноженные на 10 для покрытых льдом дорог. Снег будет где-то посередине, но где?, так что будьте осторожны при вождении любые из этих неблагоприятных условий вождения.

Позже на этой странице я использовал это данные для построения графиков и расчетов, касающихся тормозного пути до скорость и кинетическая энергия автомобиля.


ВЕРХ СТРАНИЦЫ и субиндекс


(б) Как рассчитать дистанцию ​​мышления и тормозной путь от скорости — графики времени

Графики 1a

Вы, наверное, уже встречались с графиками скорости и времени, поэтому вы должны знать, что область под частью графика скорость-время равно пройденному расстоянию на этом участке (в единицах м / с x s = m).

Графики предполагают одну и ту же машину и водителя. так что замедление при максимальном торможении такое же, поэтому отрицательный градиент — это одно и то же значение на обоих графиках.

График слева от 1a показывает начальную ситуацию водителя быстрее реагирует на поездку на более низкой скорости .

Прямоугольная область A1 = начальная скорость v1 x время реакции t1 = расстояние мышления

Площадь A1 равна расстоянию мышления, то есть расстояние, которое проезжает автомобиль, за время, необходимое водителю, чтобы реагирует на ситуацию и начинает тормозить.

Прямоугольный треугольник A2 = x начальная скорость v1 x время торможения t2 = тормозной путь

Площадь A2 — это тормозной путь, то есть расстояние, на которое транспортное средство движется от максимальной начальной скорости, когда начинается торможение, пока не останавливается.

Общая площадь = A1 + A2 = остановка расстояние

График справа от 1a показывает более медленную реакцию водителя и автомобиль движется с большей скоростью .

Это означает, что были учтены два фактора. изменено, чтобы подчеркнуть, насколько легко и драматично тормозной путь увеличено .

Итак, v2> v1 и времена t1 и t2 равны увеличивается, поэтому увеличиваются как области A1, так и A2.

Области, заштрихованные фиолетовым цветом, указывают на увеличение расстояние мышления A1 и тормозной путь A2.

Это может означать отсутствие ухода и внимание e.грамм. устал и не зацикливаясь на скоростном режиме.

Прямоугольная область A1 = начальная скорость v2 x время реакции t1 = расстояние мышления

Прямоугольный треугольник A2 = x начальная скорость v2 x время торможения t2 = тормозной путь

Итак, обе области A1 и A2 сильно увеличена, увеличивая вероятность аварии при вождении беспечно!

Общая площадь = A1 + A2 = остановка расстояние, а намного больше, чем до .

Если вы следовали вышеуказанному логические аргументы, вы сможете интерпретировать графики, если только один факторов изменилось.


ВЕРХ СТРАНИЦЫ и субиндекс


(c) Факторы, влияющие на расстояние мышления (в конечном итоге влияет и на тормозной путь)

Скорость — это первый очевидный фактор.

Чем быстрее ты , тем дальше вы будете путешествовать с тем же самым «лучшим» временем реакции, которое вы можете управлять, тем больше дистанция мышления, с которой вы ничего не можете о.

Чем длиннее ваш время реакции , тем больше ваша расстояние мышления.

Вы можете свести это к минимуму, только будучи полностью бдительными и способен реагировать так быстро, как только может ваше тело.

Последствия усталости и алкоголя повлияют на ваше бдительность и увеличить время отклика и дистанцию ​​мышления.

Есть и другие факторы.

Вы принимаете лекарства, может повлиять на вашу бдительность?

Вы отвлекаетесь на просмотр / размышления? о чем-то другом, кроме предстоящей дороги?

Ты с кем-нибудь разговариваешь? в машине дети глупые?

Даже легальное использование мобильного телефона с ручным набором, все еще потенциально отвлекает.

Плохая видимость напр. туман или дым, задержит обнаружение опасности и реакцию на нее, поэтому эффективно увеличивая время на размышления.


ВЕРХ СТРАНИЦЫ и субиндекс


(d) Факторы, влияющие на тормозной путь (в конечном итоге влияет и на тормозной путь)

Опять же, скорость — первый очевидный фактор.

Чем быстрее вы едете тем больше кинетической энергии должно быть удалено из кинетической накопитель энергии. При постоянной скорости торможения потребуется больше времени. большая скорость, потому что больше кинетической энергии должно быть преобразовано в тепло энергия в тормозной колодке и дисковой системе.

Это показано справа (тормозные колодки P контактируют с диск D).

Все факторы, обсуждаемые здесь, становятся особенно имеет решающее значение в случае экстренного торможения , или вы внезапно обнаружите Сам слишком близко к машине впереди .

Чем больше ваша скорость, тем больше вы останавливаетесь расстояние и большее расстояние, которое вы должны разрешить между одним транспортным средством и другой, например, расстояние в два шеврона для скорости 70 миль в час, которое вы видите на некоторых участки автострады.

Какими бы хорошими ни были тормоза, их нет. хорошо быть слишком близко к другому транспортному средству, т. е. в пределах остановки расстояние, если вы хотите избежать аварии, если впереди идущий автомобиль экстренный тормоз или транспортный поток быстро останавливается!

Ограничение скорости — это не просто снижение скорости, они также о сокращении тормозного пути там, где выше скорость считается опасной для определенного участка дороги.Этот для безопасности участников дорожного движения и пешеходов, например 20 миль / ч в узком улицы в застроенных районах, где может быть много людей ходьба и пересечение дорог.

Дорога состояние и погода : Неблагоприятное состояние дороги уже было упомянуто. При сухой дороге (и шинах в хорошем состоянии) вы получите максимальное сцепление с дорогой от контакта шины с дорожным покрытием при торможении, давая вам минимальное пройденное расстояние — минимальное расстояние для размышлений.Если дорога мокрая от дождя, покрыта снегом или льдом, сцепление с дорогой ослаблено. пониженный (лед> снег >> стоячая вода, все приводит к заносу на торможение). Современные шины очень хорошо тормозят, если дорога немного мокрая. и никакой очевидной стоячей воды — где можно получить «аквапланирование» / «аквапланирование» когда вы скользите по слою воды на дорожном покрытии. Листья и расколотое масло также уменьшите трение между шиной и дорогой. Все эти условия уменьшить трение шины на дороге и увеличить время торможения и тормозной путь

Состояние шин : Шины предназначены для обеспечивают максимальное сцепление с дорогой и удаляют воду из-под шин на мокрой дороге дороги.Если шины изношены (лысый или небольшой протектор), сцепление ухудшается. и жизненно важная функция трения и вытеснения воды для замедления транспортного средства уменьшаются и, таким образом, увеличивают тормозной путь и вероятность Тяжелая . Кроме того, в шинах должно быть достаточно воздуха, чтобы обеспечить правильный рабочее давление.

Эффективность тормозов : Если тормоза не в хорошем состоянии, функция торможения может быть нарушена. Тормозные колодки могут быть изношенная или негерметичная гидравлическая тормозная система может быть источником торможения обесценение.Сбалансированы ли тормоза, чтобы вы замедляли движение по прямой? — это касается и состояния шин.


ВЕРХ СТРАНИЦЫ и субиндекс


(e) Графический анализ тормозного пути, скорости и кинетической энергии движущегося автомобиля

видеть расчеты

Диаграмма ПОЗ. : KE = кинетическая энергия ( Дж, ), м = масса ( кг, ), u = начальная скорость ( м / с ), v = конечная скорость ( м / с ), с = скорость ( м / с )

a = ускорение или замедление ( м / с 2 ), Вт = работа сделано ( J ), F = усилие ( N ), d = расстояние ( м )

График 1б

График 1b выше принимает дистанцию ​​обдумывания, торможение данные о расстоянии и тормозном пути и отображают их в зависимости от типичной скорости дорожного транспортного средства.

Очевидно, все расстояния увеличиваются с увеличением скорость, но обратите внимание на два других очень важных момента.

Обратите внимание …

(i) два графика изгибаются вверх , так что «разгонного» влияния скорости на тормозной путь и в целом тормозной путь (последнее происходит из-за увеличения тормозной расстояние).

Тормозной путь и торможение расстояние не пропорционально скорости, и, что особенно важно, тормозной путь пропорционален квадрату скорости .Это означает тормозной путь увеличивается быстрее, чем увеличивается скорость.

например удвоение скорости увеличивает тормозной путь в 4 раза (2 ==> 2 2 = 4) и трехкратная скорость увеличивает тормозной путь в девять раз (3 ==> 3 2 = 9).

Расстояние мышления примерно пропорционален скорости , график ~ линейный и не изгиб вверх.Это потому, что ваше время ответа, если оно полностью бдительно, довольно постоянна, поэтому, если ваша скорость удвоится, вы просто будете вдвое больше далеко за то же время отклика.

(ii), и если вы внимательно изучите график или данные, вы видно, что удвоение скорости увеличивает тормозной путь в четыре раза.

Это означает удвоение вашего скорость, примерно увеличивает тормозной путь в 4 раза, очевидно кое-что, о чем нужно помнить, чем быстрее вы едете.

Удвоение скорость увеличивает тормозной путь в четыре раза, а скорость в три раза увеличивает его девять раз! (см. НАПОМИНАНИЕ ниже)

Это обсуждается далее и связано с формулой для кинетической энергии KE = mv 2 .

Удвоив скорость, вы увеличите кинетической энергии автомобиля, следовательно, вы в четыре раза увеличили кинетическую энергию автомобиля. энергия, которая должна быть снята при торможении (потому что KE v 2 ).См. Графики 2 и 3 и примечания ниже.

Следовательно, при удвоении скорости для постоянного тормозного усилия вам нужно удалить в четыре раза больше KE и потребуется в четыре раза большее расстояние, чтобы удалить его.

Подробнее о кинетической энергии расчеты см. Кинетический расчеты накопителя энергии

Вопрос, чтобы проиллюстрировать некоторые из идеи выше и используя приведенную ниже таблицу.

При движении со скоростью 20 миль / ч водитель расстояние мышления составляет 6,0 м, а тормозной путь — 6,0 м.

(а) Какой тормозной путь?

тормозной путь = расстояние мышления + тормозной путь = 6,0 + 6,0 = 12,0 м

(b) Оценить общий тормозной путь на скорости 40 миль в час (масштаб 2).

Если расстояние мыслей 6 м на 20 миль в час, это будет вдвое больше, чем на скорости 40 миль в час, 6 x 40/20 = 12 м.

Из аргумента KE и KE v2 тормозной путь увеличивается пропорционально квадрату масштабного коэффициента.

Так тормозной путь 6 x 2 2 = 24 м

Следовательно, тормозной путь равен 12. + 24 = 36 мес. (см. график)

(c) Оценить общий тормозной путь на скорости 80 миль в час (масштабный коэффициент 4).

Если расстояние мыслей 6 м на 20 миль в час, это будет в четыре раза больше, чем на скорости 40 миль в час, 6 x 80/20 = 24 м

Тормозной путь увеличивается на квадрат масштабного коэффициента.

Так тормозной путь 6 x 2 4 = 96 м

Следовательно, тормозной путь равен 24 + 96 = 120 м (нет на графике)


ВЕРХ СТРАНИЦЫ и субиндекс


(f) Подробнее о физике торможение автомобиля и кинетическая энергия

В механический процесс торможения в первую очередь зависит от трения между тормозами колодка и стальной диск (показан справа).Когда вы нажимаете педаль тормоза гидравлический система толкает колодки на поверхность диска , вызывая работу должно быть выполнено из-за сил сопротивления между поверхностями.

Результирующий эффект трения передает энергию от накопитель кинетической энергии автомобиля в накопитель тепловой энергии торможения система, которая в конечном итоге рассеивается в накопитель энергии окружающей среды.

трение вызывает нагрев тормозов — тормозные колодки и диск должны быть способны выдерживать высокие температуры — оба изготовлены из тугоплавких сплавов.

Немного KE теряется как звук.

Если колеса колеса буксуют на дороге, трение будет генерировать тепловую энергию, а дорога и шины увеличатся в температура.

В конце концов вся кинетическая энергия дорожный транспорт рассеивается в накопитель тепловой энергии окружение.

Итак, когда работа выполняется между тормозами и колесом кинетическая энергия дисков преобразуется в тепловую / тепловую энергию.

Чем быстрее автомобиль едет, тем больше у него запаса кинетической энергии и больше работы должно быть сделано, чтобы остановить машину.

Это также означает, что необходимо большее усилие. применяется для остановки транспортного средства при определенном торможении / остановке расстояние.

Чем больше тормозное усилие, тем больше замедление.

Сильное замедление может быть опасным, поскольку тормоза могут перегреваться, что влияет на их действие И вероятность заноса гораздо выше, особенно если дорожное покрытие скользкое из-за уже описанных условий.

Чтобы рассмотреть вопрос о кинетической энергии в контексте, изучите график 2 ниже.

График 2

График 2 показывает, как кинетическая энергия дорожное транспортное средство (например, автомобиль массой 1200 кг) меняется в зависимости от его скорости.

Вы можете увидеть, что, удвоив скорость, вы в четыре раза увеличиваете кинетическую энергию автомобиля, следовательно, вы в четыре раза увеличиваете кинетическая энергия снимается при торможении.

Это потому, что KE = mv 2 . Его скорость 2 термин, придающий этому решающее математическое значение.

При условии равномерного замедления и равномерного уменьшение скорости уменьшения кинетической энергии, означает торможение расстояние зависит от кинетической энергии и скорости 2 . Видеть график 3 сейчас.

График 3 показывает линейную зависимость между кинетическими энергия автомобиля и тормозной путь (с использованием данных правил дорожного движения Великобритании и автомобиля массой 1200 кг).

График 3

Это результат KE = mv 2 и данные о тормозном пути предполагает равномерное замедление и равномерное снижение скорости снижения кинетическая энергия за счет трения тормозов.

Как уже было сказано, торможение расстояние увеличивается быстрее скорости.

Все работы по остановке дороги транспортное средство равно начальной максимальной кинетической энергии транспортного средства.

Работы по остановке транспортного средства = всего KE транспортного средства = тормозная сила x тормозной путь

W = F x d = KE = mv 2 (в двух словах!)

W = работа в J до остановки, и вся работа выполняется за счет тормозов (при условии отсутствия заноса) через трение от накопителя KE транспортных средств к накопителю тепловой энергии тормоза и окружающая среда

F = тормозное усилие в Н (предполагается, что быть постоянным для тормозов автомобиля),

d = тормозной путь в м, м = масса автомобиля в кг, v = скорость автомобиля в м / с

При заносе на сухой дороге, резина, оставленная на дороге, говорит о том, что шины немного пошатнулись тормозной работы тоже!

Если предположить постоянное тормозное усилие (максимальное нажатие на педаль тормоза), и поскольку кинетическая энергия автомобиля равна пропорционально скорости 2 , то тормозной путь равен пропорциональна начальной кинетической энергии автомобиля.

Вот какая работа проделана уравнение говорит для постоянной тормозной силы:

KE BD и график тоже.

Дополнительное последствие: если ваша машина полна людей или грузовик полностью загружены, то кинетическая энергия при заданном скорость больше, чем если бы в транспортном средстве находился только водитель. Следовательно, при наличии дополнительной массы в транспортном средстве следует допускать дополнительное расстояние. для вашего тормозного пути из-за дополнительной кинетической энергии .

Примеры т тип. массы для дорожных транспортных средств :

вагонов 1000-1500 кг; большой фургон / одноэтажный автобус ~ 9 000 -10 000 кг; груженый ~ 30 000 — 40 000 кг.


ВЕРХ СТРАНИЦЫ и субиндекс


(грамм) Проблемы здоровья и безопасности, связанные с столкновениями с участием автотранспортных средств и велосипедистов

(мотоциклы, автомобили, грузовики, автобусы и др.)

Введение

Большое замедление (быстрое замедление вниз) предметов (аварии автомобилей или людей, падающих и ударяющихся о землю) требует значительных усилий и, очевидно, может привести к травмам.

Почему? Большие замедления требуют большого резистивная сила. Вспомните уравнение 2-го закона движения Ньютона …

F = ma , для создания большого разгон а , нужно относительно большое усилие F , независимо от массы м ,

также, чем больше масса м , тем больше усилие F необходимо для данного замедления.

В принципе, сила, испытываемая объект можно уменьшить, уменьшив замедление («более медленное» замедление вниз).

Напоминание: ускорение = изменение скорость / затраченное время, a = ∆v / ∆t , увеличить ∆t, чтобы уменьшить

С точки зрения импульса вы пытаетесь изменить импульс в течение как можно более длительного времени, чтобы минимизировать силу вовлеченный.

В следующем разделе мы применим эти идеи разработать меры безопасности, которые увеличивают время столкновения — время от первоначальное столкновение объекта с препятствием на пути к остановившемуся объекту (∆t в терминах приведенных выше уравнений), т.е. уменьшить скорость замедления.

Вам нужно знать о таких вещах, как воздух сумки и ремни безопасности в автомобилях, зоны деформации спереди и сзади автомобилей, защитные шлемы для езды на велосипеде.

Применение физики сил к расчету безопасности

При столкновении дорожного транспортного средства с неподвижный объект нормальные контактные силы между ними вызовут работа предстоит сделать.

Столкновение вызовет выделение энергии. передается из накопителя кинетической энергии транспортного средства в несколько других источников энергии магазины.

Тепловая энергия (ударное трение) и запасы упругой потенциальной энергии (эффект «хрустящего») двух объектов будут увеличится, и часть кинетической энергии перейдет в звук.

Когда все успокоится после авария, теоретически, весь запас кинетической энергии движущегося транспортного средства в конечном итоге приводит к увеличению запаса тепловой энергии в окружающей среде.

Вы можете встроить в конструкцию элементы безопасности. дорожных транспортных средств и, при необходимости, защитной одежды.

В большинстве случаев вы пытаетесь замедлить замедление — увеличить время столкновения или поглотить кинетическая энергия любого быстрого замедления и тем самым минимизировать силу a переживания тела человека. Быстрый удар вызывает резкое замедление — гораздо больше, чем даже при экстренном торможении.

Все дело в минимизации травм люди в быстро меняющейся ситуации движения .

С точки зрения физики, все о поглощение энергии удара и увеличение времени торможения — минимизация а в F = ma !

Из 2-го закона движения Ньютона: F = ma , поэтому для данной массы m , если можно сделать a замедление меньше , тормозящая сила F также уменьшен до и сводит к минимуму удары по телу и травмы.

Ремень безопасности снижает силу воздействия замедление.

При столкновении или экстренном торможении ремень безопасности немного растягивается, увеличивая время замедления и уменьшая силу вашего опыт тела против ремня безопасности. Скорость изменения импульса равна уменьшенный ( F = ∆mv / ∆t )

Быстродействующие подушки безопасности, смягчают ваше тело от сильного удара они также увеличивают время торможения и уменьшают силу ваше тело переживает.Опять же, скорость изменения импульса снижается ( F = ∆mv / ∆t )

Подушки безопасности быстро расширяются, а затем сжимаются. когда в него врезается водитель автомобиля.

Сжатие длится дольше, чем если вы врезались в приборную панель разбитой машины, или даже если вы слишком зажат ремнем безопасности.

Кузов автомобиля может иметь зоны деформации, сборки в дизайн кузова автомобиля, как спереди, так и сзади, чтобы поглотить кинетическая энергия любого сильного удара.Это увеличивает время замедления, тем самым уменьшая силу, которую испытывает ваше тело.

Фотографии (подделки) умеренно резкое столкновение автомобиля с кирпичной стеной дает представление о том, что такое «зона деформации» — это все о.

Вы увидите подобное повреждение сзади вашей машины (2-я зона деформации), если кто-то наезжает на вас сзади.

Велошлемы и защитные шлемы

Шлемы, которые носят велосипедисты или мотоциклисты наездники (мотоциклисты) имеют внутреннюю подкладку из пены (или другой энергетической поглощающий материал) для смягчения головы при ударе.

Пена увеличивает время до того, как ваша голова перестанет двигаться из-за удара.

меньшее замедление в течение большего периода времени снижает силу удара, которую испытывает ваша голова.

ВЕЛОСИПЕДНЫЕ ШЛЕМЫ

Все разработано с учетом безопасности (и комфорта).

Основная Характеристики безопасности мотоциклетного защитного шлема — это твердая защитная внешняя оболочка и «мягкий» вкладыш, поглощающий энергию удара. Комфортная набивка из пеноматериала. поглотит кинетическую энергию при ударе.

Изображение из

КАЛИФОРНИЯ ПРОГРАММА БЕЗОПАСНОСТИ МОТОЦИКЛИСТОВ

и при поддержке Калифорнийского дорожного патруля

Схема советует мотоциклистам в шлемах которые не соответствуют всем проиллюстрированным конструктивным характеристикам безопасности, должны поменять шлем!

На прогулке наткнулся на пара мотоциклистов, любезно разрешившая мне сфотографировать.Оба пережили серьезная авария, но как только защитный шлем оказался в ситуации удара, его необходимо заменить. Вы можете четко увидеть все функции, описанные в диаграмма выше.

Итак, мотоциклисты-подростки, покупайте самые безопасные шлем, он может стоить дороже, но без лучшего шлема он может стоить вам даже больше.

Исследования постоянно развиваются новые материалы для повышения эффективности функций безопасности, будь то автомобильные кузова или шлемы.

Те же идеи применимы к безопасности в игре зоны для детей и безопасности в таких видах спорта, как гимнастика

Игровое оборудование установлено на безопасность коврики, поглощающие силу удара при падении на них ребенка.

Они должны быть из резины или поролона. материалы.

Идея этой «мягкой» пьесы полы должны увеличить время воздействия за счет использования материала, который сжимается при ударе, чего не может случиться с твердой поверхностью.

Если гимнасткам нужно совершить приземление из куска устройство, которым они должны приземлиться на мягкой поверхности, чтобы уменьшить удар заставьте ноги испытать и избежать травм.

Коврики безопасности особенно необходимы, когда изучение новых процедур, в которых с большей вероятностью могут возникнуть ошибки и несчастные случаи. случаться.

Как на соревнованиях, так и на тренировках использование матов теперь является обязательным на большинстве мероприятий, и гимнастки могут использовать дополнительные мат для приземления, без вычетов, пока они приземляются в пределах указанного расстояние.

Даже футболиста носят скромные накладки на голень чтобы защитить свои ноги от жестких подкатов!

Толстый слой материала поглощает энергию удар ногой или ботинком «отлавливающего», увеличивая время удара и уменьшение силы удара.


ВЕРХ СТРАНИЦЫ и субиндекс


(h) Некоторые расширенные расчеты тормозной силы и кинетической энергии

Диаграмма ПОЗ. : KE = кинетическая энергия ( Дж, ), м = масса ( кг, ), u = начальная скорость ( м / с ), v = конечная скорость ( м / с ), с = скорость ( м / с )

a = ускорение или замедление ( м / с 2 ), Вт = проделанная работа ( J ), F = усилие ( N ), d = расстояние ( м )

1 квартал Предположим, что автомобиль массой 1200 кг движется со скоростью 18 м / с (~ 40 миль в час) и должен пройти аварийная остановка с опасностью в 30 м впереди.

(a) Рассчитайте замедление автомобиль и (б) задействованное тормозное усилие .

(a) Сначала используйте уравнение движения v 2 — u 2 = 2ad для расчета замедления.

где v = конечная скорость, u = начальная скорость, a = ускорение (∆v / ∆t), d = пройденное расстояние

Предполагая равномерное замедление и v = 0 (доходит до остановка), u = 18 м / с, d = 30 м

v 2 — u 2 = 2ad, 0 — 18 2 = 2 х а х 30

60a = -324, поэтому a = -324/60 = -5.4 м / с 2 (обратите внимание на отрицательный знак замедления)

(Это проще сделать, если у вас учитывая время торможения, можно просто использовать a = ∆v / ∆t, что я сделал в предыдущем разделе, сравнивая автомобиль и грузовой автомобиль, и назвал его 2 квартал)

(b) Затем вы используете уравнение 2-го закона Ньютона. F = ma ,

где F = замедляющее тормозное усилие, m = масса автомобиля,

а = замедление автомобиля = изменение скорости / затраченное время

Подставляя в уравнение (и можно игнорировать знак ускорения здесь, а НЕ вверху)

F = ma = 1200 x 5.4 = 6480 N

Комментарий: Вот почему ваше тело выбрасывается вперед. В замедление составляет чуть более половины значения ускорения, которое вы опыт из-за гравитационного поля Земли. Если ты при высокоскоростном ударе сила может быть намного больше и следовательно, разрушительно для вас и для машины!

См. Раздел на характеристики безопасности автомобильного транспорта

Q2 Небольшой отечественный автомобиль весом 1000 кг (1 тонна) с двумя осями на скорости 60 миль в час (26.84 м / с)

будет иметь кинетическую энергию = 0,5 мВ 2 = x 1000 х 26,84 2 = 3,6 x 10 5 J (360 кДж, 3 с.ф.)

Тяжелый седельный тягач из 6 человек оси могут весить с полной нагрузкой до 43000 кг (43 тонны) на скорости 60 миль в час. (26,84 м / с)

будет иметь кинетическую энергию = 0,5 мВ 2 = x 43000 x 26,84 2 = 1.55 х 10 8 Дж (15 500 кДж, 3 н.д.)

Теперь обе эти машины должны быть возможность остановиться на таком же безопасном расстоянии в аварийной ситуации.

Двухосный вагон будет иметь четыре комплекта тормозных колодок.

Шестиосный грузовой автомобиль будет иметь двенадцать комплектов тормозных колодок, в три раза больше, чем у автомобиля.

Это означает остановиться в такой же безопасности расстояние, тормозное усилие, прилагаемое каждым комплектом колодок в товарах Автомобиль должен быть намного больше, чем для автомобиля.

При скорости 50 миль в час (22,37 м / с) предположим, что безопасный тормозной путь — 38 м.

Затем мы можем подсчитать общую тормозное усилие необходимо для остановки через три секунды.

(я) для обоих автомобилей замедление a = ∆v / ∆t = 22,37 / 3 = 7,457 м / с 2

(ii) F = ma из 2-го закона Ньютона, сила в ньютонах, масса в кг, замедление в метров в секунду 2

Для автомобиля: F = 1000 х 7.457 = 7 460 N (3 н.ф.),

то есть Тормозное усилие 1865 Н на комплект из четырех тормозных колодок.

Для товаров автомобиль: F = 43 000 x 7,457 = 321 000 N (3 н.ф.).

то есть Тормозное усилие 26750 Н на комплект тормозных колодок.

Это означает Тормозные колодки для грузовых автомобилей должны создавать тормозное усилие более чем в 14 раз. из машины.

(Для тех знатоков в физике дорожных транспортных средств, я ценю, что они упрощены расчеты)

Подробнее о расчетах F = ma видеть Второй закон Ньютона Движение и расчет импульса

Q3 Предположим автомобиль, движущийся со скоростью 30 м / с (~ 70 миль в час), должен сделать аварийную остановку, чтобы избежать опасность.

Если масса автомобиля составляет 1500 кг, то тормозное усилие автомобиля 6000 Н и усталых водительских время реакции — 1.5 секунд, рассчитайте следующее:

(a) Рассчитайте мышление расстояние водителя (s = скорость (м / с), d — расстояние (м), t = время (s))

s = d / t, d = s x t = 30 x 1,5 = 45 м = мышление расстояние

(b) Рассчитайте начальную кинетическую энергия автомобиля (m = масса автомобиля в кг, v = скорость автомобиля (м / с)

KE = mv 2 = 0.5 х 1500 х 30 2 = 675000 = 6,75 x 10 5 Дж = начальная КЭ автомобиля

(c) Рассчитать тормозной путь для остановки автомобиля (W = выполненная работа торможения (J), d = торможение расстояние (м)

Работа, выполняемая при торможении автомобиля, должна равны кинетической энергии автомобиля (см. График 3 обсуждение)

W = F x d = KE = mv 2 = 6.75 x 10 5 Дж

W = F x d, d = W / F = 6,75 х 10 5 /6000 = 113 м = тормозной путь (3 с.ф.)

(d) Рассчитайте тормозной путь автомобиля

тормозной путь = мышление расстояние + тормозной путь

= 45 + 113 = 158 м = тормозной путь

Q4 См. реакция время эксперимент

Q5 Автомобиль с полноприводным двигателем весом 1500 кг, путешествующий в возрасте 18 лет.0 м / с (~ 40 миль / ч) съезжает с дороги, не снижая скорости до столкновения и снос кирпичной стены.

Если на снос потребовалось 0,200 секунды стена, вычислите следующие

(а) Какова начальная кинетическая энергия машины?

KE = mv 2 = 0,5 x 1500 х 18 2 = 243 000 = 2,43 х 10 5 J

(б) Какие работы выполняются на стене и машина при остановке машины?

2.43 х 10 5 J , потому что вся кинетическая энергия автомобиля должна быть удаленный.

(c) Что происходит с кинетической энергия автомобиля после удара?

Накопитель кинетической энергии автомобиль снижается до нуля и энергия преобразуется в тепло (сжатием или трением) и некоторой звуковой энергией (которая закончится вверх как тепло тоже). Так накопитель тепловой энергии стены, автомобиля и окружающий воздух увеличен .

(d) Рассчитать ставку замедление

Замедление = изменение скорости / затраченное время = ∆v / ∆t = (0-18) / 0,2 = -90 м / с 2

(e) Что такое тормозящая сила, действующая на автомобиль?

От Ньютона 2-й закон: F (N) = m (кг) x a (m / s 2 )

замедление сила = 1500 х -90 = 135 000 = -1.35 х 10 5

Сила (от стены) отрицательный, потому что он действует в противоположном направлении. направление движения автомобиля.

Если бы машина была торможение вовремя, замедляющая сила будет положительной (в каждом смысл слова!).

Q6 Представьте себе машину 1000 кг при движении со скоростью 20 м / с при аварийной остановке на расстоянии 25 м — тормозной путь.

Рассчитать среднее тормозное усилие производится водителем при нажатии на педаль тормоза.

Для решения этого вопроса используйте несколько формул.

(a) Рассчитайте кинетическую энергию машина.

KE = 0,5 мВ 2 = 0,5 x 1000 x 202 = 200 000 Дж

(b) Какие работы необходимо сделать, чтобы машина остановилась? Поясните свой ответ.

Если кинетическая энергия автомобиля составляет 200000 Дж, то 200 000 Дж работы должны быть выполнены, чтобы довести KE автомобиля до нуля, т.е. нулевая скорость.

(c) Рассчитайте среднее торможение требуется сила.

Работа (Дж) = сила (Н) x расстояние (м)

работа = 200 000 Дж и торможение дистанция 25 м

сила = работа / расстояние = 200 000 / 25 = Среднее тормозное усилие 8000 Н.

Q7 Массовый фургон 2000 кг отклоняется от дороги со скоростью 30 м / с и становится неподвижным после наезда каменная стена.

(a) Если сила удара на фургон 48 000 Н, рассчитайте время остановки.

F = m∆v / ∆t , заменяя

48 000 = 2000 х (30-0) / ∆t

48 000 = 60 000/ ∆t

∆t = 60 000/48 000 = 1.25 с

(b) Объясните, как ремень безопасности и надувание подушки безопасности может спасти жизнь водителю.

При ударе тело водителя разогнался вперёд.

(i) Ремень безопасности растягивается достаточно, чтобы уменьшить скорость изменения количества движения — увеличение времени замедления.

(ii) «Мягкая» надутая подушка безопасности. также снижает скорость изменения количества движения и поглощает кинетические энергия при столкновении с телом водителя.

Q8 A 20000 кг дорога автомобиль приходит к аварийной остановке.

Равномерное тормозное усилие 8000 Н применяется водителем до тех пор, пока транспортное средство не остановится в расстояние 20 м.

(a) Рассчитайте скорость автомобиль незадолго до того, как были задействованы тормоза.

Работа при торможении = тормозная сила x расстояние задействованных тормозов = 8000 x 20 = 160000 Н

Всего работ выполнено в торможение = кинетическая энергия транспортного средства в момент сначала включаются тормоза.

KE = 0,5 мВ 2 , перестановка дает v = √ {(KE / (0,5 x m)}

v = √ {(160 000 / (0,5 x 20 000)} = 4 м / с

(б) Каковы основные энергоносители? происходит передача магазина?

Кинетическая энергия автомобиль в основном преобразуется за счет трения, чтобы увеличить накопитель тепловой энергии частей автомобиля и окружающей среды воздушный или автомобильный.

Подробнее о кинетической энергии расчеты см. Кинетический расчеты накопителя энергии


ВЕРХ СТРАНИЦЫ и субиндекс


(i) Простая реакция время экспериментов

Но может сопровождаться умеренно сложные расчеты!

Время вашей реакции на ситуацию обычно может быть 0.2 к 0,8 секунды при полной готовности. Однако на время вашей реакции могут повлиять усталость, плохое самочувствие, наркотики, алкоголь, другими словами все, что влияет на скорость работы вашего мозга.

См. Введение к нервной системе, включая рефлекторную дугу

Вы можете провести довольно простые эксперименты, чтобы проверить свой время реакции на ту или иную ситуацию. Однако, поскольку время реакции слишком короткий, секундомер бесполезен, но есть способы измерить ваш время реакции косвенно путем проведения других измерений, из которых вы можете рассчитайте время своей реакции.

(a) Экран компьютера — где вы как можно быстрее отвечаете на что-то появляется на экране.

В этой ситуации компьютер программное обеспечение генерирует что-то на экране и автоматически ваш ответ, отслеживая ваш контакт с клавиатурой или щелкнув мышью.

Я быстро написал чрезвычайно простая компьютерная программа для проверки вашей реакции на появление X на экран.

Время отклика test: вероятно, работает только на платформах Microsoft, и может не все?

Ваша антивирусная защита может запросить его, потому что это файл .exe , но он написан с составлен BBC BASIC и не должен представлять никакой угрозы. К сожалению, Я так и не научился писать на многоплатформенном профессиональном компьютере язык программирования, но мне не хватает проектов для веб-сайтов!

(b) Простой тест на физическую реакцию — падение линейка для испытания на падение

Вы заставляете кого-то держать линейку вертикально , с большой и указательный пальцы над чужой рукой, готовый поймать большим и указательным пальцами.

Первое изображение справа. В линейку следует держать наверху шкалы и твердыми руками от оба человека.

Ловящий человек должен иметь середина их большого пальца и палец примыкают к нулю на см шкала — присядьте, чтобы убедиться, что вы читаете шкалу по горизонтали.

Тогда, без предупреждения, человек, держащий линейку, отпусти это. Второй человек должен отреагировать как можно быстрее и поймать упавшую линейку большим и указательным пальцами.

Второе изображение справа. Чем больше расстояние, тем медленнее ваша реакция!

Когда поймают, вы читаете, как далеко линейка упала, считая показания с точностью до сантиметра, откуда находятся середина их большого пальца и пальца.

Вы повторяете эксперимент номер раз, чтобы получить среднее значение , но это не особенно точное эксперимент.

У вас должны быть устойчивые руки, а не пусть линейка раскачивается или падает под углом, отличным от вертикального. Ты должны также использовать ту же линейку и те же люди, которые роняют линейку и ловить его (критерии честного тестирования), хотя, очевидно, можно сравнить результаты одного человека с другим.

Чем меньше время отклика, тем далее правитель падает до того, как его поймают. Вы можете повторить поэкспериментируйте, отвлекая фон — группу людей разговариваете поблизости, или кто-то пытается вовлечь вас в разговор или Музыка.

Q4 Затем вы можете сделать несколько «изящных» вычисления, чтобы на самом деле получить реальное время отклика — так что вы использование косвенных данных для получения времени отклика.

Он включает двухэтапный расчет.

Предположим, что правитель пойман после среднее падение 25 см.

(i) Вы используете уравнение v 2 — u 2 = 2ad , для расчета конечной скорости (подробнее расчеты по этому уравнению)

v = конечная скорость (м / с), u = начальная скорость (м / с), a = ускорение = 9.8 м / с 2 (ускорение свободного падения),

и d = пройденное расстояние (м)

Так как u = 0 и d = 25/100 = 0,25 м

v 2 — 0 = 2 x 9,8 x 0,25 = 4,9

v = √4.9 = 2,214 м / с (ее не так точно, но мы оставим н.ф. до конца)

(ii) Теперь мы можем использовать ускорение формула для расчета времени отклика.

а = ∆v / ∆t, где a = ускорение (9,8 м / с 2 ), ∆v = изменение скорость (м / с) и ∆t = время отклика

Следовательно: 9,8 = 2,214 / ∆t, ∆t = 2,214 / 9,8 = 0,23 с (2 н.ф.)

Итак, дальше В среднем время отклика составило около четверти секунды.


ВЕРХ СТРАНИЦЫ и субиндекс


Движение и связанные силы отмечает индекс (включая Законы Ньютона Движение)

1.Скорость и скорость — взаимосвязь между расстояние и время, графики расстояние-время gcse Physics

2. Ускорение, интерпретация и расчеты графика скорость-время. решение проблем Примечания к редакции физики gcse

3. Ускорение, трение, эффекты сопротивления и эксперименты с конечной скоростью Примечания к редакции физики gcse

4. Первый, второй и третий законы Ньютона. Расчет движения, инерции и F = ma Примечания к редакции физики gcse

5.Время реакции тормозной путь и пример расчеты Примечания к редакции физики gcse

6. Упругие и неупругие столкновения, импульс. вычислений и 2-го закона Ньютона движение заметки gcse по физике



Версия IGCSE заметки тормозной путь скорость торможения кинетическая энергия KS4 физика научные заметки на тормозной путь скорость торможения кинетическая энергия руководство по физике GCSE заметки по тормозному пути скорость торможения кинетическая энергия для школ колледжи академии научные курсы репетиторы изображения рисунки диаграммы для тормозного пути скорость торможения кинетическая энергия наука пересмотр примечания на тормозной путь скорость торможения кинетическая энергия для пересмотра модулей физики разделы физики заметки для помощи в понимании тормозной путь скорость торможения кинетическая энергия университетские курсы физики карьера в науке и физике вакансии в машиностроении технический лаборант стажировка инженер стажировка по физике США 8 класс 9 класс 10 AQA Примечания к редакции GCSE 9-1 по физике, тормозной путь скорость торможения кинетическая энергия GCSE примечания по тормозному пути скорость торможения кинетическая энергия Edexcel GCSE 9-1 физика наука пересмотр примечания к тормозной путь скорость торможения кинетическая энергия для OCR GCSE 9-1 21 век физика научные заметки о тормозном пути скорость торможения кинетическая энергия OCR GCSE 9-1 Шлюз физики примечания к изменениям тормозного пути Скорость торможения кинетическая энергия WJEC gcse science CCEA / CEA gcse science

ВЕРХ СТРАНИЦЫ и субиндекс

AQA | Тематическое содержание | Компонент 5 — Физика: энергия, силы и структура материи

Студенты должны знать и понимать следующее содержание.

Содержание Дополнительные указания и предлагаемые ТДА Ссылка на спецификацию GCSE Combined Science: Trilogy Ссылка на спецификацию GCSE Combined Science: Synergy
Результат 6

Скорость измеряется расстоянием, пройденным за определенное время.

Единицы скорости включают метры в секунду и километры в час.

Простые вычисления средней скорости по формуле: скорость = потребуется расстояние / время.

Рекомендуемое действие для TDA Изучите скорость тележка меняется при скатывании по склону.

6.5.4.1.2

4.7.1.1

Результат 7

Тормозной путь транспортного средства — это сумма расстояний, транспортное средство движется во время реакции водителя (мышление расстояние) и расстояние, которое он проходит под действием тормозной силы (тормозной путь).

Для заданного тормозного усилия, чем больше скорость автомобиля, тем больше больше тормозной путь.

Студенты могут найти полезным ознакомиться с Правилами дорожного движения.

6.5.4.3.1

4.7.1.10

Результат 8

Время реакции варьируется от человека к человеку.Типичные значения варьируются от От 0,2 до 0,9 с.

Знание и понимание методов, используемых для измерения реакции человека раз.

Знание того, как на время реакции водителя может повлиять усталость, наркотики и алкоголь. Отвлекающие факторы также могут повлиять на способность водителя реагировать.

Студенты должны уметь интерпретировать и оценивать измерения простые методы измерения разного времени реакции студенты.

Студенты должны уметь оценивать влияние различных факторов на расстояние мышления.

Предложено активность для TDA Изучите факторы, влияющие на человека время реакции, например, усталость, отвлечение, упражняться.

6.5.4.3.2

4.2.1.6

Результат 9

На тормозной путь транспортного средства могут повлиять неблагоприятные дороги и погодные условия и плохое состояние автомобиля.

Студенты должны уметь анализировать данную ситуацию, чтобы определить, как это может повлиять на торможение.

К неблагоприятным дорожным условиям относятся влажные или гололедицы. Плохое состояние автомобиля ограничивается тормозами или шинами автомобиля.

6.5.4.3.3

4.7.1.10

Physics Investigation: Влияние скорости на тормозной путь — GCSE Science

Выдержки из этого документа…

Physics Investigation:

Влияние скорости на тормозной путь

Hashim Al-Hasani 11R

Intro

Скорость — это пройденное расстояние за каждый момент / единицу заданного времени. Скорость, расстояние и время связаны друг с другом, что показано в следующем уравнении, которое можно изменить, чтобы найти формулу для каждого измерения:

Скорость (м / с) = Расстояние (м) ÷ Время (с)

Тормозной путь объекта — это расстояние, необходимое для замедления объекта, пока он не остановится (остановится).Объект станет неподвижным только тогда, когда движущая сила противодействует, например трению и сопротивлению воздуха.

Кинетическая энергия объекта — это энергия, которую он получает в результате своего движения. Уравнение кинетической энергии:

Какие факторы могут повлиять на тормозной путь объекта?

Ну, есть 3 основных фактора, которые влияют на тормозной путь объекта:

Высота, с которой объект выпущен

Это может повлиять на тормозной путь, поскольку чем выше объект, когда он выпущен, тем больше его исходный GPE.По мере того, как объект продолжает движение вниз, его GPE уменьшается, а его кинетическая энергия увеличивается на такое же количество потерянной потенциальной энергии гравитации.

… подробнее.

450

50 м

Считывание

Скорость

Расстояние

м / с

мм

1

0,71633

343

2

0.70225

0003000371023

320

4

0,71225

320

5

0,72254

348

40 м

Чтение

Скорость

0,79

325

2

0,74184

280

3

0,74627

331

4

0,73746

293

5

0,75301

м / с

мм

1

1.12108

730

2

1.1062

719

3

1.12613

760

4

1.11607

749

0003

0003

0003

0003

0003

0003

0003

0003

Расстояние

м / с

мм

1

1.05485

646

2

1.04167

644

3

1.04167

67604603

658

5

1.04167

659

70 м

Считывание

Скорость

Расстояние

м / с

мм

1

000

000 0,957 570

3

0,9434

569

4

0,94697

581

5

0,95057

581

Высота опускания (см)

9000 с4 Средняя скорость (м)

9000 с4 )

90

0.23

41

80

0,46

134

70

0,61

219

60

0,74

310

50

0,71

0,71

30

0,95

573

20

1,05

657

10

1,12

736

90

3

90

3

90

0,59524

30m

Расстояние

м / с

мм

1

0.59524

212

2

0,62344

214

3

224

5

0,62657

240

Расстояние

м / с

мм

1

0.45537

137

2

0,4562

133

3

3

0,46642

131

5

0,46041

134

Чтение

Скорость

Расстояние

м / с

мм

9 124802

39

2

0,22543

40

3

3

3

0,25

0,22957

43

5

0,21758

43

РЕЗУЛЬТАТЫ MARBLE

3

66см

Вес = 4,2 г (1 п.п.)

… подробнее.

Уверенность в заключении

В целом я вполне уверен в своем заключении. Я получил ряд результатов, которые убедительно свидетельствовали о том, что скорость мяча влияет на разрывную дистанцию ​​мяча. Хотя, как показывают большинство графиков, многие из полосок диапазона немного велики, а это означает, что результаты не очень надежны из-за большого разброса данных, возможно, это было из-за изгиба рампы при катании по мячу или что световые ворота были размещены неправильно в то время, что объясняет причину, по которой его результаты были так разбросаны.

На протяжении всего эксперимента я получил много выбросов в моих результатах, как показано в двух таблицах. Это могло произойти по любой из следующих причин:

  • В некоторых повторах положение светового затвора не было в том же месте, что и в других повторах, или поэтому

Несмотря на эти выбросы, большинство средних значений были близки к кривой наилучшего соответствия или находились на ней, поэтому после исключения выбросов результаты в целом были надежными.

…Подробнее.

Эта письменная работа студента — одна из многих, которые можно найти в нашем разделе GCSE Forces and Motion.

Физика — Тормозной путь автомобиля

Физика — Тормозной путь автомобиля

Следующая таблица основана на данных, первоначально опубликованных в Popular Science и AutoWeek журналы 1 .Тормозной путь для новых автомобилей. (1991–1995). Значения тормозного пути (от 60 миль / ч = 97 км / ч = 27 м / с) для 55 автомобилей в диапазоне от 114 футов (35 м) до 179 футов (55 м) в среднем 140 футов (43 м).

Автомобиль

Тормозной путь
от 60 миль / ч (97 км / ч)

Замедление

футов

метра

фут / с 2

м / с 2

БМВ М3

120

37

32.3

9,8

Toyota Celica GT

128

39

30,2

9,2

Линкольн Континенталь

131

40

29.6

9,0

Nissan Maxima

142

43

27,3

8,3

Шевроле Блейзер

158

48

24.5

7,5

Додж Кольт GL

167

51

23,2

7,1

Транспортные средства длиной от 13 до 13.От 4 футов (4,1 м) до 20,3 футов (6,2 м) в среднем 15,7 футов (4,8 м).


Следующие данные были взяты из Indiana Drivers Manual , Бюро транспортных средств (дата публикации не найдена).

Автомобиль

Средний тормозной путь на скорости 55 миль / ч (включая реакцию время)

Легковой автомобиль

190 футов.

Седельный тягач (груженый) с крутыми тормозами

256 футов.

Седельный тягач (груженый) с горячими тормозами

430 футов.

Седельный тягач (пустой)

249 футов.

Только трактор (бобтейл)

243 фут.

1 Источник : Р. К. Никлин, Kinematics Tailgating, in Учитель физики, том 35, февраль 1997 г., стр. 78


последнее обновление 22 марта 2000 г., автор: JL Stanbrough

Онлайн-конвертеры единиц измерения

Случайный преобразователь

Онлайн-конвертеры единиц измерения

Конвертер длины и расстоянияКонвертер массыКонвертер объёма сухого воздуха и общих измерений при варкеПреобразователь площадиПреобразователь объёма и общего измерения при варкеПреобразователь температурыПреобразователь давления, напряжения, модуля ЮнгаПреобразователь энергии и работыПреобразователь силыПреобразователь силыКонвертер времениЛинейный конвертер скорости и скоростиКонвертер угловой эффективностиПреобразователь топливной эффективности, расхода топлива и информации о расходе топливаКонвертер единиц Хранение данныхКурсы обмена валютЖенская одежда и размеры обувиМужская одежда и размеры обувиКонвертер угловой скорости и частоты вращенияКонвертер ускоренияКонвертер углового ускоренияКонвертер плотностиКонвертер удельного объемаПреобразователь момента инерцииПреобразователь момента силыКонвертер крутящего моментаПреобразователь удельной энергии, теплоты сгорания (на единицу температуры на массу) Конвертер удельной энергии Преобразователь интерваловКонвертер коэффициента теплового расширенияПреобразователь теплового сопротивленияПреобразователь теплопроводности Конвертер удельной теплоемкости ter Конвертер скорости передачиКонвертер уровня звукаКонвертер чувствительности микрофонаКонвертер уровня звукового давления (SPL) Конвертер уровня звукового давления с выбираемым эталонным давлениемКонвертер яркостиКонвертер яркостиКонвертер яркостиКонвертер разрешения цифрового изображенияПреобразователь частоты и длины волныОптическая мощность (диоптрия) в преобразователь фокусного расстоянияПреобразователь оптической мощности (диоптрий) в увеличение (X) Конвертер электрического заряда Конвертер плотности зарядаКонвертер поверхностной плотности зарядаКонвертер объёмной плотности заряда Преобразователь электрического токаЛинейный преобразователь плотности токаПреобразователь плотности поверхностного токаПреобразователь напряженности электрического поляПреобразователь электрического потенциала и напряженияПреобразователь электрического сопротивленияКонвертер электрического сопротивленияПреобразователь электрической проводимостиПреобразователь электрической проводимостиПреобразователь емкостиПреобразователь индуктивностиПреобразователь реактивной мощности переменного токаПреобразователь единиц магнитного поля в ваттах и ​​дБм Конвертер плотности потока Конвертер мощности поглощенной дозы излучения, Конвертер мощности дозы полного ионизирующего излученияРадиоактивность.Преобразователь радиоактивного распада Преобразователь радиационного воздействияРадиация. Конвертер поглощенной дозы Конвертер метрических префиксов Конвертер передачи данных Конвертер единиц типографии и цифрового изображения Конвертер единиц измерения объема древесиныКалькулятор молярной массыПериодическая таблица

Этот онлайн-конвертер единиц измерения позволяет быстро и точно переводить многие единицы измерения из одной системы в другую. Страница преобразования единиц представляет собой решение для инженеров, переводчиков и для всех, чья деятельность требует работы с величинами, измеренными в различных единицах.

Вы можете использовать этот онлайн-конвертер для преобразования нескольких сотен единиц (включая метрическую, британскую и американскую) в 76 категорий или нескольких тысяч пар, включая ускорение, площадь, электрическую энергию, энергию, силу, длину, свет, массу, массовый расход, плотность, удельный объем, мощность, давление, напряжение, температура, время, крутящий момент, скорость, вязкость, объем и емкость, объемный расход и многое другое.
Примечание: Целые числа (числа без десятичной точки или показателя степени) считаются точными до 15 цифр, а максимальное количество цифр после десятичной точки равно 10.», То есть« умноженное на десять в степени ». Электронная нотация обычно используется в калькуляторах, а также учеными, математиками и инженерами.

Преобразователи общих единиц

Конвертер длины и расстояния : метр, километр, сантиметр, миллиметр, нанометр, ярд, фут, дюйм, парсек, световой год, астрономическая единица, расстояние до Луны (от Земли до Луны), лига , миля, морская миля (международная), сажень, длина кабеля (международная), точка, пиксель, калибр, планковская длина…

Конвертер массы : грамм, килограмм, миллиграмм, тонна (метрическая), фунт, унция, камень (США), камень (Великобритания), карат, зерно, талант (библейский греческий), драхма (библейский греческий), денарий (библейский римлянин), шекель (библейский иврит), масса Планка, масса протона, атомная единица массы, масса электрона (покой), масса Земли, масса Солнца …

Сухой объем и стандартные измерения при приготовлении пищи : литр, бочка сухой (США), пинта сухой (США), квартовый сухой (США), peck (США), peck (Великобритания), bushel (США), bushel (UK), cor (библейский), homer (библейский), ephah (библейский) ), seah (библейский), omer (библейский), cab (библейский), log (библейский), кубометр.

Конвертер площади : миллиметр², сантиметр², метр², километр², гектар, акр, дюйм², фут², ярд², миля², сарай, круглый дюйм, поселок, роуд, стержень², окунь², усадьба, шест², сабин, арпент, куерда, квадратная верста, квадратный аршин, квадратный фут, квадратный сажень, площадь Планка …

Конвертер объёма и общепринятых единиц измерения температуры : метр³, километр³, миллиметр³, литр, гектолитр, миллилитр, капля, бочка (масло), бочка (США) ), баррель (Великобритания), галлон (США), галлон (Великобритания), кварта (США), кварта (Великобритания), пинта (США), пинта (Великобритания), баррель (нефть), баррель (США), баррель (Великобритания ), галлон (США), галлон (Великобритания), кварта (США), кварта (Великобритания), пинта (США), пинта (Великобритания), ярд³, фут³, дюйм³, регистровая тонна, 100 кубических футов…

Конвертер температуры : кельвин, градус Цельсия, градус Фаренгейта, градус Ренкина, градус Реомюра, температура Планка.

Преобразователь давления, напряжения, модуля Юнга : паскаль, килопаскаль, мегапаскаль, миллипаскаль, микропаскаль, нанопаскаль, атмосферно-техническая, стандартная атмосфера, ksi, psi, ньютон на метр², бар, миллибар, килограмм-сила / метр², грамм- сила / сантиметр², тонна-сила (короткая) / фут², фунт-сила / фут², миллиметр ртутного столба (0 ° C), дюйм ртутного столба (32 ° F), сантиметр водяного столба (4 ° C), фут водяного столба (4 ° C) , метр морской воды…

Конвертер энергии и работы : джоуль, килоджоуль, мегаджоуль, миллиджоуль, мегаэлектронвольт, электрон-вольт, эрг, киловатт-час, мегаватт-час, ньютон-метр, килокалория (IT), калория (пищевая), Британские тепловые единицы (IT), мега Btu (IT), тонна-час (охлаждение), тонна нефтяного эквивалента, баррель нефтяного эквивалента (США), мегатонна, тонна (взрывчатые вещества), килограмм в тротиловом эквиваленте, дин-сантиметр, грамм-сила-сантиметр, килограмм-сила-метр, килопонд-метр, фут-фунт, дюйм-фунт, энергия Планка …

Преобразователь мощности : ватт, киловатт, мегаватт, милливатт, лошадиные силы, вольт-ампер, ньютон-метр / секунда, джоуль / секунда, мегаджоуль в секунду, килоджоуль в секунду, миллиджоуль в секунду, джоуль в час, килоджоуль в час, эрг в секунду, британские тепловые единицы (IT) в час, килокалорий (IT) в час…

Преобразователь силы : ньютон, килоньютон, миллиньютон, дин, джоуль / метр, джоуль / сантиметр, грамм-сила, килограмм-сила, тонна-сила (короткая), кип-сила, килопунт-сила, фунт-сила сила, унция-сила, фунтал, фунт-фут / секунда², pond, sthene, grave-force, миллиграв-сила …

Преобразователь времени : секунда, миллисекунда, наносекунда, пикосекунда, минута, час, день, неделя, месяц, год, декада, век, тысячелетие, планковское время, год (юлианский), год (високосный), год (тропический), год (сидерический), год (григорианский), две недели, встряска…

Конвертер линейной скорости и скорости : метр / секунда, километр / час, километр / секунда, миля / час, фут / секунда, миля / секунда, узел, узел (Великобритания), скорость света в вакууме, космический скорость — первая, космическая скорость — вторая, космическая скорость — третья, скорость Земли, скорость звука в чистой воде, Мах (стандарт СИ), Мах (20 ° C и 1 атм), ярд / секунду …

Угол Преобразователь : градус, радиан, град, гон, минута, секунда, знак, мил, оборот, круг, поворот, квадрант, прямой угол, секстант.

Конвертер топливной экономичности, расхода топлива и экономии топлива : метр / литр, километр / литр, миля (США) / литр, морская миля / литр, морская миля / галлон (США), километр / галлон (США), литр / 100 км, галлон (США) / миля, галлон (США) / 100 миль, галлон (Великобритания) / миля, галлон (Великобритания) / 100 миль …

Конвертер чисел : двоичный, восьмеричный, десятичный, шестнадцатеричный, основание-3, основание-4, основание-5, основание-6, основание-7, основание-9, основание-10, основание-11, основание-12, основание-13, основание-14, основание-15, основание-20, основание-21, основание-22, основание-23, основание-24, основание-28, основание-30, основание-32, основание-34, основание-36…

Преобразователь единиц информации и хранения данных : бит, байт, слово, четверное слово, MAPM-слово, блок, килобит (10³ бит), кибибит, кибибайт, килобайт (10³ байтов), мегабайт (10⁶ байтов), гигабайт (10⁹ байтов), терабайт (10¹² байтов), петабайт (10¹⁵ байтов), эксабайт (10¹⁸ байтов), гибкий диск (3,5 ED), гибкий диск (5,25 HD), Zip 250, Jaz 2 ГБ, CD (74 минут), DVD (2 слоя 1 сторона), диск Blu-ray (однослойный), диск Blu-ray (двухслойный) …

Курсы обмена валют : евро, доллар США, канадский доллар, британский фунт стерлингов, японская иена, швейцарский франк, аргентинское песо, австралийский доллар, бразильский реал, болгарский лев, чилийское песо, китайский юань, чешская крона, датская крона, египетский фунт, венгерский форинт, исландская крона, индийская рупия, индонезийская рупия, новый израильский шекель , Иорданский динар, малазийский ринггит, мексиканское песо, новозеландский доллар, норвежская крона, пакистанская рупия, филиппинское песо, румынский лей, российский рубль, саудовский риял, сингапурский доллар, Южноафриканский рэнд, южнокорейский вон, шведская крона, новый тайваньский доллар, тайский бат, турецкая лира, украинская гривна…

Размеры женской одежды и обуви : женские платья, костюмы и свитера, женская обувь, женские купальные костюмы, размер буквы, бюст, дюймы, естественная талия, дюймы, заниженная талия, дюймы, бедра, дюймы, бюст, сантиметры, Натуральная талия, сантиметры, Заниженная талия, сантиметры, Бедра, сантиметры, Длина стопы, мм, Торс, дюймы, США, Канада, Великобритания, Европа, континентальный, Россия, Япония, Франция, Австралия, Мексика, Китай, Корея ..

Размеры мужской одежды и обуви : мужские рубашки, мужские брюки / брюки, размер мужской обуви, размер букв, шея, дюймы, грудь, дюймы, рукав, дюймы, талия, дюймы, шея, сантиметры, грудь, сантиметры, Рукав, сантиметры, Талия, сантиметры, Длина стопы, мм, Длина стопы, дюймы, США, Канада, Великобритания, Австралия, Европа, континентальный, Япония, Россия, Франция, Италия, Испания, Китай, Корея, Мексика…

Механика

Преобразователь угловой скорости и частоты вращения : радиан / секунда, радиан / день, радиан / час, радиан / минута, градус / день, градус / час, градус / минута, градус / секунда, оборот / день, оборот / час, оборот / минута, оборот / секунда, оборот / год, оборот / месяц, оборот / неделя, градус / год, градус / месяц, градус / неделя, радиан / год, радиан / месяц, радиан / неделя.

Преобразователь ускорения : дециметр / секунда², метр / секунда², километр / секунда², гектометр / секунда², декаметр / секунда², сантиметр / секунда², миллиметр / секунда², микрометр / секунда², нанометр / секунда², пикометр / секунда², фемтометр / секунда² , аттометр в секунду², галлон, галилей, миля в секунду², ярд в секунду², фут в секунду², дюйм / секунду², ускорение свободного падения, ускорение свободного падения на Солнце, ускорение свободного падения на Меркурии, ускорение свободного падения на Венере , ускорение свободного падения на Луне, ускорение свободного падения на Марсе, ускорение свободного падения на Юпитере, ускорение свободного падения на Сатурне…

Конвертер плотности : килограмм / метр³, килограмм / сантиметр³, грамм / метр³, грамм / сантиметр³, грамм / миллиметр³, миллиграмм / метр³, миллиграмм / сантиметр³, миллиграмм / миллиметр³, экзаграмма / литр, петаграмм / литр, тераграмма / литр, гигаграмм / литр, мегаграмм / литр, килограмм / литр, гектограмм / литр, декаграмм / литр, грамм / литр, дециграмм / литр, сантиграмм / литр, миллиграмм / литр, микрограмм / литр, нанограмм / литр, пикограмм / литр , фемтограмм / литр, аттограмм / литр, фунт / дюйм³ …

Конвертер удельного объема : метр³ / килограмм, сантиметр³ / грамм, литр / килограмм, литр / грамм, фут³ / килограмм, фут³ / фунт, галлон (США ) / фунт, галлон (Великобритания) / фунт.

Преобразователь момента инерции : килограмм-метр², килограмм-сантиметр², килограмм-миллиметр², грамм-сантиметр², грамм-миллиметр², килограмм-сила-метр-секунда², унция-дюйм², унция-сила-дюйм-секунда², фунт-фут², фунт-сила-фут-секунда², фунт-дюйм². , фунт-сила-дюйм-секунда², ударный фут².

Конвертер момента силы : метр ньютон, метр килоньютон, метр миллиньютон, метр микроньютон, метр тонна-сила (короткий), метр тонна-сила (длинный), метр тонна-сила (метрический), метр килограмм-сила, грамм-сила-сантиметр, фунт-сила-фут, фунт-фут, фунт-дюйм.

Гидротрансформатор : ньютон-метр, ньютон-сантиметр, ньютон-миллиметр, килоньютон-метр, дин-метр, дин-сантиметр, дин-миллиметр, килограмм-сила-метр, килограмм-сила-сантиметр, килограмм-сила-миллиметр, грамм-сила-метр, грамм- сила-сантиметр, грамм-сила-миллиметр, унция-сила-фут, унция-сила-дюйм, фунт-сила-фут, фунт-сила-дюйм.

Термодинамика — тепло

Конвертер удельной энергии, теплоты сгорания (на массу) : джоуль / килограмм, килоджоуль / килограмм, калория (IT) / грамм, калория (th) / грамм, Btu (IT) / фунт, BTU (th) / фунт, килограмм / джоуль, килограмм / килоджоуль, грамм / калория (IT), грамм / калория (th), фунт / BTU (IT), фунт / Btu (th), фунт / лошадиная сила-час, грамм / лошадиная сила (метрическая) -час, грамм / киловатт-час.

Конвертер удельной энергии, теплоты сгорания (на объем) : джоуль / метр³, джоуль / литр, мегаджоуль / метр³, килоджоуль / метр³, килокалория (IT) / метр³, калория (IT) / сантиметр³, терм / фут³, терм / галлон (Великобритания), британские тепловые единицы (IT) на фут³, британские тепловые единицы на фут³, CHU / фут³, метр³ / джоуль, литр / джоуль, галлон (США) / лошадиная сила-час, галлон (США) / лошадиная сила (метрическая система) )-час.

Конвертер теплопроводности : ватт / метр / K, ватт / сантиметр / ° C, киловатт / метр / K, калория (IT) / секунда / сантиметр / ° C, калория (th) / секунда / сантиметр / ° C , килокалория (IT) / час / метр / ° C, килокалория (th) / час / метр / ° C, BTU (IT) дюйм / секунда / фут² / ° F, BTU (th) дюйм / секунда / фут² / ° F , BTU (IT) фут / час / фут² / ° F, Btu (th) фут / час / фут² / ° F, BTU (IT) дюйм / час / фут² / ° F, BTU (th) дюйм / час / фут² / ° F.

Конвертер удельной теплоемкости : джоуль / килограмм / K, джоуль / килограмм / ° C, джоуль / грамм / ° C, килоджоуль / килограмм / K, килоджоуль / килограмм / ° C, калория (IT) / грамм / ° C, калория (IT) / грамм / ° F, калория (th) / грамм / ° C, килокалория (IT) / килограмм / ° C, килокалория (th) / килограмм / ° C, килокалория (IT) / килограмм / K , килокалория (th) / килограмм / K, килограмм-сила-метр / килограмм / K, фунт-сила-фут / фунт / ° R, Btu (IT) / фунт / ° F, Btu (th) / фунт / ° F, Btu (IT) / фунт / ° R, Btu (th) / фунт / ° R, Btu (IT) / фунт / ° C, CHU / фунт / ° C.

Конвертер плотности теплового потока : ватт / метр², киловатт / метр², ватт / сантиметр², ватт / дюйм², джоуль / секунда / метр², килокалория (IT) / час / метр², килокалория (IT) / час / фут², калория (IT) / минута / сантиметр², калория (IT) / час / сантиметр², калория (th) / минута / сантиметр², калория (th) / час / сантиметр², дина / час / сантиметр, эрг / час / миллиметр², фут-фунт / минута на фут², лошадиные силы на фут², лошадиные силы (метрические единицы) на фут², британские тепловые единицы (IT) / секунда на фут², британские тепловые единицы (IT) в минуту на фут², британские тепловые единицы (ИТ) на час / фут², британские тепловые единицы (единицы) / секунда на дюйм² , Btu (th) / секунда / фут², Btu (th) / минута / фут², Btu (th) / час / фут², CHU / час / фут².

Конвертер коэффициента теплопередачи : ватт / метр² / K, ватт / метр² / ° C, джоуль / секунда / метр² / K, килокалория (IT) / час / метр² / ° C, килокалория (IT) / час / фут² / ° C, BTU (IT) / секунда / фут² / ° F, Btu (th) / секунда / фут² / ° F, BTU (IT) / час / фут² / ° F, BTU (th) / час / фут² / ° F, CHU / час / фут² / ° C.

Гидравлика — жидкости

Конвертер объемного расхода : метр³ / секунда, метр³ / день, метр³ / час, метр³ / минута, сантиметр³ / день, сантиметр³ / час, сантиметр³ / минуту, сантиметр³ / секунда, литр / день, литр в час, литр в минуту, литр в секунду, миллилитр в день, миллилитр в час, миллилитр в минуту, миллилитр в секунду, галлон (США) в день, галлон (США) в час, галлон (США) в минуту, галлон (США) в секунду, галлон (Великобритания) в день, галлон (Великобритания) в час, галлон (Великобритания) в минуту, галлон (Великобритания) в секунду, килобаррель (США) в день, баррель (США) в день…

Конвертер массового расхода : килограмм / секунда, грамм / секунда, грамм / минута, грамм / час, грамм / день, миллиграмм / минута, миллиграмм / час, миллиграмм / день, килограмм / минута, килограмм / час , килограмм / день, экзаграмм / секунда, петаграмма / секунда, тераграмма / секунда, гигаграмма / секунда, мегаграмм / секунда, гектограмм / секунда, декаграмма / секунда, дециграмма / секунда, сантиграмма / секунда, миллиграмм / секунда, микрограмм / секунда, тонна (метрическая) / секунда, тонна (метрическая) / минута, тонна (метрическая) / час, тонна (метрическая) / день …

Конвертер молярной скорости потока : моль / секунда, экзамен / секунда, петамоль / секунда, терамоль в секунду, гигамоль в секунду, мегамоль в секунду, киломоль в секунду, гектомоль в секунду, декамоль в секунду, децимоль в секунду, сантимоль в секунду, миллимоль в секунду, микромоль в секунду, наномоль в секунду, пикомоль в секунду, фемтомоль в секунду. секунда, аттомоль в секунду, моль в минуту, моль в час, моль в день, миллимоль в минуту, миллимоль в час, миллимоль в день, километр в минуту, километр в час, километр в день.

Преобразователь потока массы : грамм / секунда / метр², килограмм / час / метр², килограмм / час / фут², килограмм / секунда / метр², грамм / секунда / сантиметр², фунт / час / фут², фунт / секунда / фут².

Конвертер молярной концентрации : моль / метр³, моль / литр, моль / сантиметр³, моль / миллиметр³, километр / метр³, километр / литр, километр / сантиметр³, километр / миллиметр³, миллимоль / метр³, миллимоль / литр, миллимоль / сантиметр³, миллимоль / миллиметр³, моль / дециметр³, молярный, миллимолярный, микромолярный, наномолярный, пикомолярный, фемтомолярный, аттомолярный, зептомолярный, йоктомолярный.

Конвертер массовой концентрации в растворе : килограмм / литр, грамм / литр, миллиграмм / литр, часть / миллион, гран / галлон (США), гран / галлон (Великобритания), фунт / галлон (США), фунт / галлон (Великобритания), фунт / миллион галлон (США), фунт / миллион галлон (Великобритания), фунт / фут³, килограмм / метр³, грамм / 100 мл.

Конвертер динамической (абсолютной) вязкости : паскаль-секунда, килограмм-сила-секунда на метр², ньютон-секунда на метр², миллиньютон-секунда на квадратный метр, дин-секунда на сантиметр², равновесие, экзапуаз, петапуаз, терапуаз, гигапуаз, мегапуаз, килопуаз, гектопуаз, декапуаз, деципуаз, сантипуаз, миллипуаз, микропуаз, наноуаз, пикопуаз, фемтопуаз, аттопуаз, фунт-сила-секунда / дюйм², фунт-сила-секунда / фут², фунт-секунда / фут², грамм / сантиметр / секунда…

Конвертер кинематической вязкости : метр² / секунда, метр² / час, сантиметр² / секунда, миллиметр² / секунда, фут² / секунда, фут² / час, дюйм² / секунда, стоксы, экзастоки, петастоки, терастоки, гигастоксы, мегастоксы, килостоки, гектостоки, декастоки, децистоки, сантистоки, миллистоки, микростоки, наностоки, пикостоки, фемтостоки, аттостоки.

Преобразователь поверхностного натяжения : ньютон на метр, миллиньютон на метр, грамм-сила на сантиметр, дина на сантиметр, эрг / сантиметр², эрг / миллиметр², фунт на дюйм, фунт-сила / дюйм.

Акустика — звук

Преобразователь чувствительности микрофона : децибел относительно 1 вольт на 1 паскаль, децибел относительно 1 вольта на 1 микропаскаль, децибел относительно 1 вольта на 1 дин на квадратный сантиметр, децибел относительно 1 вольт на 1 микробар, вольт на паскаль, милливольт на паскаль, микровольт на паскаль.

Преобразователь уровня звукового давления (SPL) : ньютон на квадратный метр, паскаль, миллипаскаль, микропаскаль, дин / квадратный сантиметр, бар, миллибар, микробар, уровень звукового давления в децибелах.

Фотометрия — свет

Конвертер яркости : кандела на метр², кандела на сантиметр², кандела на фут², кандела на дюйм², килокандела на метр², стильб, люмен на метр² / стерадиан, люмен на сантиметр² / стерадиан² / люмен на фут². стерадиан, нит, миллинит, ламберт, миллиламберт, фут-ламберт, апостиль, блондель, брил, скот.

Конвертер силы света : кандела, свеча (немецкий язык), свеча (Великобритания), десятичная свеча, свеча (пентан), пентановая свеча (мощность 10 свечей), свеча Хефнера, единица измерения яркости, десятичный буж, люмен / стерадиан, свеча (Международный).

Конвертер освещенности : люкс, метр-свеча, сантиметр-свеча, фут-свеча, фот, nox, кандела стерадиан на метр², люмен на метр², люмен на сантиметр², люмен на фут², ватт на сантиметр² (при 555 нм) .

Преобразователь частоты и длины волны : герцы, эксагерцы, петагерцы, терагерцы, гигагерцы, мегагерцы, килогерцы, гектогерцы, декагерцы, децигерцы, сантигерцы, миллигерцы, микрогерцы, единицы, микрогерцы / наночастицы , длина волны в петаметрах, длина волны в тераметрах, длина волны в гигаметрах, длина волны в мегаметрах, длина волны в километрах, длина волны в гектометрах, длина волны в декаметрах…

Конвертер оптической силы (диоптрии) в фокусное расстояние : Оптическая сила (диоптрическая сила или преломляющая сила) линзы или другой оптической системы — это степень, в которой система сходится или рассеивает свет. Он рассчитывается как величина, обратная фокусному расстоянию оптической системы, и измеряется в инверсных метрах в СИ или, чаще, в диоптриях (1 диоптрия = м⁻¹)

Электротехника

Конвертер электрического заряда : кулон, мегакулон , килокулон, милликулон, микрокулон, нанокулон, пикокулон, абкулон, EMU заряда, статкулон, ESU заряда, франклин, ампер-час, миллиампер-час, ампер-минута, ампер-секунда, фарадей (на основе углерода 12), элементарный плата.

Преобразователь электрического тока : ампер, килоампер, миллиампер, биот, абампер, ЭДС тока, статампер, ЭДС тока, СГС э.м. единица, CGS e.s. единица, микроампер, наноампер, ток Планка.

Линейный преобразователь плотности тока : ампер / метр, ампер / сантиметр, ампер / дюйм, абампер / метр, абампер / сантиметр, абампер / дюйм, эрстед, гильберт / сантиметр, ампер / миллиметр, миллиампер / метр, миллиампер / дециметр , миллиампер / сантиметр, миллиампер / миллиметр, микроампер / метр, микроампер / дециметр, микроампер / сантиметр, микроампер / миллиметр.

Конвертер поверхностной плотности тока : ампер / метр², ампер / сантиметр², ампер / дюйм², ампер / мил², ампер / круговой мил, абампер / сантиметр², ампер / миллиметр², миллиампер / миллиметр², микроампер / миллиметр², миллиметр / миллиметр² миллиампер / сантиметр², микроампер / сантиметр², килоампер / сантиметр², ампер / дециметр², миллиампер / дециметр², микроампер / дециметр², килоампер / дециметр².

Преобразователь напряженности электрического поля : вольт на метр, киловольт на метр, киловольт на сантиметр, вольт на сантиметр, милливольт на метр, микровольт на метр, киловольт на дюйм, вольт на дюйм, вольт на мил, абвольт на сантиметр, статвольт / сантиметр, статвольт / дюйм, ньютон / кулон, вольт / микрон.

Преобразователь электрического потенциала и напряжения : вольт, милливольт, микровольт, нановольт, пиковольт, киловольт, мегавольт, гигавольт, теравольт, ватт / ампер, абвольт, EMU электрического потенциала, статвольт, ESU электрического потенциала, планковское напряжение.

Преобразователь электрического сопротивления : Ом, мегаом, микром, вольт / ампер, обратный сименс, abohm, EMU сопротивления, статом, ESU сопротивления, квантованное сопротивление Холла, импеданс Планка, миллиом, кОм.

Преобразователь удельного электрического сопротивления : омметр, ом-сантиметр, ом-дюйм, микром-сантиметр, микром-дюйм, ом-сантиметр, статом-сантиметр, круговой мил-ом / фут, ом-кв.миллиметр на метр.

Преобразователь электрической проводимости : сименс, мегасименс, килосименс, миллисименс, микросименс, ампер / вольт, mho, gemmho, micromho, abmho, statmho, квантованная проводимость Холла.

Конвертер электропроводности : сименс / метр, пикосименс / метр, mho / метр, mho / сантиметр, abmho / метр, abmho / сантиметр, статмо / метр, статмо / сантиметр, сименс / сантиметр, миллисименс / метр, миллисименс / сантиметр, микросименс / метр, микросименс / сантиметр, единица электропроводности, коэффициент проводимости, доли на миллион, шкала 700, шкала частей на миллион, шкала 500, частей на миллион, шкала 640, TDS, частей на миллион, шкала 640, TDS, части на миллион, шкала 550, TDS, частей на миллион, шкала 500, TDS, частей на миллион, шкала 700.

Преобразователь емкости : фарад, эксафарад, петафарад, терафарад, гигафарад, мегафарад, килофарад, гектофарад, декафарад, децифарад, сантифарад, миллифарад, микрофарад, емкость, нанофарад, аттофарад, фе , статфарад, ЭСУ емкости.

Конвертер индуктивности : генри, экзагенри, петагенри, терагенри, гигагенри, мегагенри, килогенри, гектогенри, декагенри, децигенри, сантигенри, миллигенри, микрогенри, наногенри, пикогенри, индуктивность, фемогенри, атогенри , статенри, ЭСУ индуктивности.

Преобразователь реактивной мощности переменного тока : реактивный вольт-ампер, реактивный милливольт-ампер, реактивный киловольт-ампер, реактивный мегавольт-ампер, реактивный гигавольт-ампер.

Американский преобразователь калибра проводов : Американский калибр проволоки (AWG) — это стандартизированная система калибра проводов, используемая в США и Канаде для измерения диаметров цветных электропроводящих проводов, включая медь и алюминий. Чем больше площадь поперечного сечения провода, тем выше его допустимая нагрузка по току.Чем больше номер AWG, также называемый калибром провода, тем меньше физический размер провода. Самый большой размер AWG — 0000 (4/0), а самый маленький — 40. В этой таблице перечислены размеры и сопротивление AWG для медных проводников. Используйте закон Ома для расчета падения напряжения на проводнике.

Магнитостатика, магнетизм и электромагнетизм

Преобразователь магнитного потока : Вебер, Милливебер, Микровебер, вольт-секунда, единичный полюс, мегалин, килолин, линия, максвелл, тесла-метр², тесла-сантиметр², гаусс-сантиметр², квант магнитного потока.

Конвертер плотности магнитного потока : тесла, Вебер / метр², Вебер / сантиметр², Вебер / дюйм², Максвелл / метр², Максвелл / сантиметр², Максвелл / дюйм², Гаусс, линия / сантиметр², линия / дюйм², гамма.

Radiation and Radiology

Конвертер мощности поглощенной дозы излучения, суммарной мощности дозы ионизирующего излучения : серый цвет в секунду, эксагрей в секунду, петагрей в секунду, тераграрей в секунду, гигаграй в секунду, мегагрей в секунду, килограмм в секунду, гектограмм / секунда, декаграй / секунда, дециграй / секунда, сантигрей / секунда, миллиграй / секунда, микрогрей / секунда, наногрей / секунда, пикграй / секунда, фемтогрей / секунда, аттогрей / секунда, рад / секунда, джоуль / килограмм / секунда, ватт на килограмм, зиверт в секунду, миллизиверт в год, миллизиверт в час, микрозиверт в час, бэр в секунду, рентген в час…

Радиоактивность. Конвертер радиоактивного распада : беккерель, петабеккерель, терабеккерель, гигабеккерель, мегабеккерель, килобеккерель, миллибеккерель, кюри, килокюри, милликюри, микрокюри, нанокюри, пикокюри, резерфорд, одна / секунда, дезинтеграции.

Конвертер облучения : кулон на килограмм, милликулон на килограмм, микрокулон на килограмм, рентген, миллирентген, микрорентген, тканевый рентген, Паркер, респ.

Радиация. Конвертер поглощенной дозы : рад, миллирад, джоуль / килограмм, джоуль / грамм, джоуль / сантиграм, джоуль / миллиграмм, серый, эксагрей, петагрей, терагрей, гигагрей, мегагрей, килограмм, гектагрей, декаграй, децигрей, сантигрей, микрогрей, миллиграм , наногрей, пикграй, фемтогрей, аттогрей, зиверт, миллизиверт, микрозиверт …

Разные преобразователи

Конвертер метрических префиксов : нет, йотта, дзетта, экса, пета, тера, гига, мега, кило, гектодека, дека , деци, санти, милли, микро, нано, пико, фемто, атто, зепто, йокто.

Конвертер передачи данных : бит в секунду, байт в секунду, килобит в секунду (SI по умолчанию), килобайт в секунду (SI по умолчанию), кибибит в секунду, кибибайт в секунду, мегабит в секунду (SI по умолчанию) , мегабайт в секунду (SI по умолчанию), мебибит в секунду, мебибайт в секунду, гигабит в секунду (SI по умолчанию), гигабайт в секунду (SI по умолчанию), гибибит в секунду, гибибит в секунду, терабит в секунду (SI по умолчанию). .), терабайт в секунду (по умолчанию SI), тебибит в секунду, тебибайт в секунду, Ethernet, Ethernet (быстрый), Ethernet (гигабит), OC1, OC3, OC12, OC24, OC48 …

Типографика и цифровой Конвертер единиц изображения : твип, метр, сантиметр, миллиметр, символ (X), символ (Y), пиксель (X), пиксель (Y), дюйм, пика (компьютер), пика (принтер), точка (DTP / PostScript) ), point (компьютер), point (принтер), en, cicero, em, Didot point.

Конвертер единиц измерения объема пиломатериалов : кубический метр, кубический фут, кубический дюйм, футы для досок, тысяча футов для досок, шнур, шнур (80 фут3), футы для шнура, узел, поддон, поперечина, стяжка.

Калькулятор молярной массы : Молярная масса — это физическое свойство, которое определяется как масса вещества, деленная на его количество в молях. Другими словами, это масса одного моля определенного вещества.

Периодическая таблица : Периодическая таблица представляет собой список всех химических элементов, упорядоченных слева направо и сверху вниз по их атомным номерам, электронным конфигурациям и повторяющимся химическим свойствам, расположенным в форме таблицы таким образом, чтобы элементы с аналогичные химические свойства отображаются в вертикальных столбцах, называемых группами.У некоторых групп есть имена, а также номера. Например, все элементы группы 1, кроме водорода, являются щелочными металлами, а элементы группы 18 — благородными газами, которые ранее назывались инертными газами. Различные строки таблицы называются периодами, потому что это расположение отражает периодическое повторение сходных химических и физических свойств химических элементов по мере увеличения их атомного номера.

Ответить

Ваш адрес email не будет опубликован. Обязательные поля помечены *